78
KAPLAN LSAT PREP LSAT RELEASED TEST XII EXPLAINED A Guide to the October, 1994 LSAT KAPLAN The answer to the test question.

12-PrepTest 12 Explsdl.keywin.org/5/5/559e448ca144e5a6335f12b48619009d.pdf · KAPLAN LSAT PREP LSAT RELEASED TEST XII EXPLAINED A Guide to the October, 1994 LSAT KAPLAN The answer

Embed Size (px)

Citation preview

Page 1: 12-PrepTest 12 Explsdl.keywin.org/5/5/559e448ca144e5a6335f12b48619009d.pdf · KAPLAN LSAT PREP LSAT RELEASED TEST XII EXPLAINED A Guide to the October, 1994 LSAT KAPLAN The answer

KAPLAN LSAT PREP

LSAT

RELEASED TEST XIIEXPLAINED

A Guide to the October, 1994 LSAT

KAPLANThe answer to the test question.

Page 2: 12-PrepTest 12 Explsdl.keywin.org/5/5/559e448ca144e5a6335f12b48619009d.pdf · KAPLAN LSAT PREP LSAT RELEASED TEST XII EXPLAINED A Guide to the October, 1994 LSAT KAPLAN The answer

1994 Stanley H. Kaplan Educational Center Ltd

All rights reserved. No part of this book may be reproduced in any form, byphotostat, microfilm, xerography or any other means, or incorporated into anyinformation retrieval system, electronic or mechanical, without the written permissionof Stanley H. Kaplan Educational Center Ltd.

Page 3: 12-PrepTest 12 Explsdl.keywin.org/5/5/559e448ca144e5a6335f12b48619009d.pdf · KAPLAN LSAT PREP LSAT RELEASED TEST XII EXPLAINED A Guide to the October, 1994 LSAT KAPLAN The answer

© K A P L A N 1

SECTION I:

LOGICAL REASONING

Page 4: 12-PrepTest 12 Explsdl.keywin.org/5/5/559e448ca144e5a6335f12b48619009d.pdf · KAPLAN LSAT PREP LSAT RELEASED TEST XII EXPLAINED A Guide to the October, 1994 LSAT KAPLAN The answer

LSAT PREP ________________________________________________________________ LSAT Test XII Explained: Section I

2 © K A P L A N

1. (D)Although the stimulus may strike you as a bit scattershot, all the evidence is assembled topersuade the reader that research efforts into making the Martian climate inhabitable arejustified. This is the main point — everything else (the possibility of successful climatechange, the success of other long term efforts, the benefits of climate research) is support.

(A) merely restates part of the evidence. Very well, the change is possible, but the author istrying to persuade us that research should be begun now.

(B) states a drawback to the author's plan, a drawback the author deals with by saying thatother worthwhile projects have taken centuries to complete.

(C) concentrates on an illustration, instead of the main point, and furthermore misses themeaning of the comparison between cathedral building and changing the Martian climate.The comparison had to do with time, not effort.

(E) again is evidence supporting the main point, giving one of the reasons for beginningthe research the author recommends.

• A common problem with wrong choices on “main point” questions is that theyrestate part of the argument, usually some of the supporting evidence. Don't besatisfied with evidence; you want a choice that expresses the purpose of the wholeargument.

• Think of the main point as a one sentence summation of the author's argument. Ifyou had to compress the whole argument into one sentence, what would it be?

2. (C)The first sentence practically demands evidence (“Why do you say that kids should votebecause adults do? What do you mean by that?”) and thus should strike us right away asthe conclusion. The evidence that follows, the reason for the policy recommendation,points to the disjunction between kids and grownups and the latter’s inability to representkids’ interests. But since the interests of kids aren’t mentioned in the conclusion, the authormust be assuming something about them, and happily only (C) mentions kids’ interests; itmust be correct. The Denial Test proves it: If, contrary to (C), there’s no reason for someoneto represent kids’ interests, then the author’s rationale for granting kids the vote flies outthe window.

(A),(D) Pretty blatant scope shifts. “All human beings” (A) covers infants, but the authorisn’t committed to the belief that toddlers should get the vote. And the argument is solelyconcerned with voting rights, not “all rights” (D).

(B) The only grownup/kid differences to which the author is committed are the ones thatrender grownups unable to fairly represent kids’ interests. Those differences might ormight not constitute “the most important” ones.

(E) The past history of adolescent voting is outside the scope of an argument whosepurpose is to tout a present-day policy.

Page 5: 12-PrepTest 12 Explsdl.keywin.org/5/5/559e448ca144e5a6335f12b48619009d.pdf · KAPLAN LSAT PREP LSAT RELEASED TEST XII EXPLAINED A Guide to the October, 1994 LSAT KAPLAN The answer

LSAT PREP ________________________________________________________________ LSAT Test XII Explained: Section I

© K A P L A N 3

• The moment you start reading a stimulus you should be in search of the statementthat is most in need of evidence. That’ll be the conclusion, the linchpin of all of yourLogical Reasoning work.

• Expect that, early in an LR section, you’ll get some relatively straightforwardquestions dealing largely with arguments’ scope.

3. (E)The Keyword “Admittedly” tells you that the phrase in question is something the author isacceding to or allowing — something, in other words, that seems to contradict or goagainst the argument. The subsequent Keyword “But” clearly signals a contrast, alerting usthat the statement in question may support his view after all. And indeed, the concededdissimilarity of kids and grownups ends up being used as evidence for allowing kids tovote: Because they’re different, grownups can’t be depended upon to represent kids’interests.

(A) Does the difference between kids and grownups make you say “Really? How so?Where’s your evidence?” Uh-uh. Besides, no phrase beginning with “Admittedly” can be aconclusion.

(B) What key word is rendered more precise? (Silence.) The statement isn’t there for thepurpose of greater definition.

(C) “Admittedly” does not mean “Consequently,” which it would have to mean in orderfor (C) to be correct.

(D) implies that the statement is sheer filler, and distracting filler at that. But (see above) thestatement does play a key evidentiary role.

• Always read and analyze Keywords, especially when a question is asking about thepurpose of a statement. Keywords only exist because they serve authors’ purposes.Here, merely interpreting the Keywords surrounding the statement in questionshould lead you to (E).

4. (B)Though low taxes, proximity to markets and proximity to raw materials are of roughlyequal importance to businesses which are relocating, businesses are willing to relocate to amunicipality with high business taxes, but not to a town that is far away from markets orraw materials. Choice (B) resolves this discrepancy by showing how a higher tax rate onbusiness generally means better education and better services, two factors which are alsovery important to business. In other words, the negative factor of higher taxes can actuallyhave positive results, while there’s no comparable benefit to being far away from rawmaterials or markets.

(A) provides useless background information. What part of a city’s tax revenue comes frombusiness has no bearing on the question.

Page 6: 12-PrepTest 12 Explsdl.keywin.org/5/5/559e448ca144e5a6335f12b48619009d.pdf · KAPLAN LSAT PREP LSAT RELEASED TEST XII EXPLAINED A Guide to the October, 1994 LSAT KAPLAN The answer

LSAT PREP ________________________________________________________________ LSAT Test XII Explained: Section I

4 © K A P L A N

(C) confuses the issue by shifting the scope of the discussion from businesses which arerelocating to a municipality to businesses which are already there.

(D) Since the discrepancy we’re asked to resolve concerns only tax-rates, proximity tomarkets, and proximity to raw materials, the issue of where to find highly-educatedworkers is irrelevant.

(E) is too vague: “Businesses have sometimes tried...” How many businesses? How manysucceeded? This choice just isn’t strong enough for us to draw a general conclusion abouthow businesses and municipalities interact.

• When asked to resolve an apparent discrepancy or paradox, look for an alternateexplanation that does not contradict the evidence expressed in the passage.

• Often, a plausible alternate explanation is suggested by something in the stimulusitself. Here, the stimulus identifies five factors of equal importance to businesses,but the “discrepancy” involves only three. Recognizing the importance of the othertwo is the key to solving this question correctly.

5. (D)Oscar argues that his unattributed use of material from Myers’ book isn’t plagiarismbecause he had obtained Myers’ permission to do so. In other words, Oscar’s claim is basedon 1) the evidence that Myers relinquished her exclusive right to the material, and 2) theassumption that this is sufficient justification for his use of the material. The principleexpressed in choice (D) restates this assumption in more general terms and thereforejustifies Oscar’s action.

(A) is a distortion based on a logical fallacy. The principle that an author has no right toquote another author without her permission is NOT equivalent to the principle that anauthor does have the right to quote another author with her permission.

(B) isn’t helpful here, because we don’t know the length of the passages Oscar used fromMyers’ work. And even if we did, this principle undermines Oscar’s argument, because hedidn’t cite Myers as the source of the passages which he quoted.

(C) is irrelevant, because Oscar took passages from “Myers’ book,” a published source.

(E) is outside the scope of the argument. Oscar didn’t borrow passages from his own work,he took them from Myers.

• On principle questions, the correct choice must be consistent with the argument’scentral assumption. In fact, it’s often just a restatement of that assumption in moregeneral terms.

• Always read the question stem before you read the stimulus. Here, you would haverealized that Millie’s side of the argument has no bearing on this question.

Page 7: 12-PrepTest 12 Explsdl.keywin.org/5/5/559e448ca144e5a6335f12b48619009d.pdf · KAPLAN LSAT PREP LSAT RELEASED TEST XII EXPLAINED A Guide to the October, 1994 LSAT KAPLAN The answer

LSAT PREP ________________________________________________________________ LSAT Test XII Explained: Section I

© K A P L A N 5

6. (A)Oscar’s defense is that Myers acquiesced to the use of her unattributed prose, to whichMillie responds that, essentially, Myers’ agreeing or disagreeing is “outside the scope.”Millie reinterprets plagiarism as (“fundamentally,” meaning at bottom) a lie that misleadsreaders, and that’s something to which the original author is not party. To “analyze” is totake apart and interpret something, and that’s what Millie does to the concept ofplagiarism — to the detriment of Oscar’s stance.

(B) Since Oscar freely admits the use of unattributed Myers prose, it would besuperfluous of Millie to present evidence to that effect.

(C) The “proof” of Myers’ permission would simply be to offer Myers’ letter forinspection, and Millie in no way hints that Oscar would be unable to do so. Millie doesn’tsay that Myers did not give permission to use her prose, but that Myers cannot givepermission to plagiarize, since [see above] the meaning of plagiarism has nothing to dowith author say-so.

(D) Millie’s issue is whether Myers’ permission constitutes an adequate defense ofplagiarism, not whether authors have “rights” prohibiting it.

(E) “Lack of credibility” may sound fancy at first reading, but has no relevance here.Millie seems to think that Oscar’s having received Myers’ permission is credible enough;she just doesn’t buy it as a defense.

• “Fundamentally” is a great Emphasis Keyword. Watch for words and phrases thatauthors use to signal what they’re most interested in: They’ll usually be the key tothe right answer as well.

7. (B)Soil scientists and home gardeners have experienced very different results when usingcompost. How to resolve the discrepancy? They’re using different kinds of compost.

(A) shifts the scope away from the argument and focuses instead on the relative credibilityof different groups of soil scientists. Since we don’t know which group, if any, the soilscientists in the argument belong to, this point is irrelevant.

(C) It may, in fact, be true that all plants need both good soil structure and essentialnutrients in order to thrive. But knowing this doesn’t help explain why gardeners’ plantscan get nutrients from compost when soil scientists say it can’t be done.

(D) only compounds the dilemma, by eliminating soil difference between scientists’ testplots and home gardens as a possible explanation of the discrepancy.

(E) eliminates a possible alternate explanation, that plants in home gardens require fewernutrients than those in the scientists’ test plots.

Page 8: 12-PrepTest 12 Explsdl.keywin.org/5/5/559e448ca144e5a6335f12b48619009d.pdf · KAPLAN LSAT PREP LSAT RELEASED TEST XII EXPLAINED A Guide to the October, 1994 LSAT KAPLAN The answer

LSAT PREP ________________________________________________________________ LSAT Test XII Explained: Section I

6 © K A P L A N

• Since the key to a paradox/discrepancy question is finding a reasonable alternateexplanation, any choice which contradicts a possible explanation, such as (D) and (E)above, should be eliminated immediately.

• Always be suspicious of the arbitrary equation of two situations, e.g. home gardenand scientists’ test plot. What differences might exist between the two that aren’tmentioned in the stimulus?

8. (C)Here's an inference question that's open to prephrasing. The average salary for allHappywell employees is lower than the average for either dieticians or physical therapists.So some Happywell employee must be earning less than either of those two averages, inorder to drag the total average down to $40,000. That's (C): At least one Happywellemployee earns less than the average for physical therapists. You can use the Denial Testfor inference questions here: If no Happywell employee earned less than $42,000, then theaverage salary for Happywell employees clearly could not be $40,000.

(A) is a false comparison — we have no information about the numbers of either dieticiansor physical therapists.

(B) claims that no single dietician made less than $42,000, but we don't know what thelowest dietician’s salary was, only the average dietician’s salary.

(D) and (E) make similar mistakes, speaking of the highest paid and lowest paid dieticianand physical therapist. We have no information about any of this; knowing the averagesalary doesn't tell you the range of salaries.

• “Average” questions try to trick you by getting you to make false assumptions aboutindividuals based on averages. Knowing the average salaries of group A and groupB doesn't automatically allow you to compare the extreme salaries of group A andgroup B.

9. (E)We get the crux of the argument in the first sentence. The conclusion, or main point, is thatone cannot depend on multinational grain companies to start the process “that wouldreform the world's food-distribution system.” Why not? (that’s the question that the firstsentence should have prompted in your mind) — Because such companies operate inorder to maximize profits. The rest of the stimulus simply explains what it means for acompany to be interested in maximizing profits (change is incidental, stability ispreferred).

(A) explains why the evidence leads to the conclusion (interest in profits discouragesinterest in change) but is not itself the main point, or conclusion.

(B) and (C) are both evidence. (B) goes back to the second sentence, restating the author'sreason for saying that economic change caused by multinational companies is merelyincidental. (C) simply repeats the main piece of evidence from the first sentence's capsule

Page 9: 12-PrepTest 12 Explsdl.keywin.org/5/5/559e448ca144e5a6335f12b48619009d.pdf · KAPLAN LSAT PREP LSAT RELEASED TEST XII EXPLAINED A Guide to the October, 1994 LSAT KAPLAN The answer

LSAT PREP ________________________________________________________________ LSAT Test XII Explained: Section I

© K A P L A N 7

statement of the argument: multinational companies are unreliable because they operate tomaximize profits.

(D) introduces a scope shift, bringing up an irrelevant issue. The author says nothing aboutwhether or not reform of the food-distribution system is needed, only about who can berelied upon to initiate such reform.

• On main point questions, reject any choice that rehashes material that is prefaced inthe stimulus by a Keyword such as “although” or “because” or “since” (like (C)above). Such Keywords signal evidence, not conclusion, so they never introduce themain point.

• When you’re looking for the conclusion but no conclusion Keywords like“therefore” or “thus” are present, ask yourself which sentence would sound best if“therefore” were appended to it. And remember: The conclusion can appear in thebeginning or middle or end of the stimulus.

10. (B)The author defines a realistic performance as one in which an actor reproduces whateverbehavior audiences associate with the emotion being experienced by the actor’s character.“Method” actors actually feel their characters’ emotions, while traditional actors justimitate the relevant behaviors. In order to conclude that audiences will consider Methodperformances more realistic, the author must assume that by actually feeling a particularemotion, an actor is more likely to reproduce the behavior audiences associate with it.Without that assumption, the argument would fall apart.

(A) shifts the scope to a comparison of which type of acting has more effect on theaudience’s emotions, something the stimulus never discussed.

(C) is irrelevant. Even if realism isn’t an essential criterion for evaluating performances, theauthor’s conclusion could still be true, provided the assumption in (B) is true.

(D) distorts the argument. The author claims only that Method actors are better atreproducing realistic behavior, not that traditional actors don’t even try to be realistic.

(E) Also irrelevant. The issue is how realistic audiences consider Method performances,not whatever prerequisites are necessary for Method actors to portray characters.

• The Denial Test can be used to check answers to assumption questions. Deny ornegate your choice and see if the argument falls apart. If it does fall apart, as it doeshere with (B), the answer is correct. If the conclusion is unaffected, as the denial of(C) shows here, the choice is wrong.

Page 10: 12-PrepTest 12 Explsdl.keywin.org/5/5/559e448ca144e5a6335f12b48619009d.pdf · KAPLAN LSAT PREP LSAT RELEASED TEST XII EXPLAINED A Guide to the October, 1994 LSAT KAPLAN The answer

LSAT PREP ________________________________________________________________ LSAT Test XII Explained: Section I

8 © K A P L A N

11. (D)Most of this stimulus is just filler. If we boil the argument down to its bare bones, it lookssomething like this:

An increasing number of former West Germans will sell their old cars. [evidence]After they sell their old cars, they’ll buy new cars. [assumption]The German new-car market will improve. [conclusion]

Choice (D) restates the assumption and therefore provides the most support to theargument.

(A) confuses the issue by shifting the scope from the German new-car market as a whole,which is the subject of the conclusion, to the car markets in former West Germany andformer East Germany.

(B) is a mild weakener. It provides a possible reason why Germans who sell their used carsmight not be able to buy new cars.

(C) provides background information which is not relevant to the argument. MostGermans don’t change cars that often. So what? The passage provides evidence that themarket conditions are right for many Germans to change cars now.

(E) The distinction between North American cars and European cars is a useless one. Thepassage is not concerned with the German car industry, only the German car market.

• To strengthen a conclusion, look for a choice that is essentially a paraphrase of theargument’s main assumption.

• The correct answer to a strengthen and weaken question must have some bearing onthe core of the argument, the basic evidence-assumption-conclusion structure.Choices that address peripheral issues should be eliminated.

12. (D)The number of people exposing themselves to the risk of melanoma began to decrease in1980. But the number of newly reported cases of melanoma rose sharply in 1982. The keyhere is the phrase “newly reported.” The number of people who had melanoma may havegone down. The number of people found to have melanoma went up. The development in1982 of a new and better way to test for melanoma provides a satisfactory explanation forthis occurrence.

(A) is useless background information. We just don’t have enough information to build aplausible alternative explanation around the age of melanoma sufferers.

(B) explains why melanoma was widespread before 1980. It doesn’t explain why 1982 saw arise in the number of melanoma cases reported.

Page 11: 12-PrepTest 12 Explsdl.keywin.org/5/5/559e448ca144e5a6335f12b48619009d.pdf · KAPLAN LSAT PREP LSAT RELEASED TEST XII EXPLAINED A Guide to the October, 1994 LSAT KAPLAN The answer

LSAT PREP ________________________________________________________________ LSAT Test XII Explained: Section I

© K A P L A N 9

(C) This choice implies that until 1982, many people exposed themselves to the sun (andthe risk of melanoma) because they mistakenly thought they needed the exposure toproduce vitamin D. However, this is directly contradicted by the passage, which states thatthe number of people sunbathing began to decrease in 1980.

(E) doesn’t help to resolve the paradox. If sunbathers since 1980 have been using effectivesunscreens, why would melanoma cases rise in 1982?

• In the world of the LSAT, there is always a way to resolve the paradox. That’s whythe word “apparent” is in the question stem. What does this mean for you? It meansif you see a choice that doesn’t provide a plausible alternate explanation that resolvesthe discrepancy, eliminate it immediately.

• The LSAT testmakers, like lawyers, use language very precisely. There is a differencebetween “crimes committed” and “crimes reported,” between “diseases suffered”and “diseases treated.” Read carefully.

13. (D)The evidence is that Minlandians, who can only get Boltese-language TV, watch less TVand do more reading for pleasure than people anywhere else. The psychologist’sexplanation for this is that people in general prefer to be entertained in their nativelanguage, but this only makes sense if some of what the Minlandians are reading is inMinlandian. If, contrary to (D), the Minlandians are all reading Boltese-language novels forfun, then they are not displaying any preference for native-language entertainment.

(A) shifts the scope. The issue is whether people prefer entertainment in their nativelanguage, not whether they get any pleasure at all from entertainment that’s not in theirnative language.

(B) introduces useless background information. The psychologist needn’t assume anythingabout how the Minlandians became fluent in Boltese, since that isn’t part of the argument.

(C)’s comparison is irrelevant. Minlandia doesn’t need the highest rate of bilingualism inthe world for its residents to prefer to be entertained in their native language.

(E) Way outside the scope. The point is whether Minlandians prefer being entertained inMinlandian, not what they do when they’re watching Boltese TV.

• In assumption questions, keep an eye out for leaps of logic between evidence andconclusion. The correct choice is the one that bridges the gap.

Page 12: 12-PrepTest 12 Explsdl.keywin.org/5/5/559e448ca144e5a6335f12b48619009d.pdf · KAPLAN LSAT PREP LSAT RELEASED TEST XII EXPLAINED A Guide to the October, 1994 LSAT KAPLAN The answer

LSAT PREP ________________________________________________________________ LSAT Test XII Explained: Section I

10 © K A P L A N

14. (D)The argument attempts to prove a conclusion about numbers based on evidence aboutproportion. The conclusion is that a greater number of full-time teachers than part-timeteachers quit in their first year. The evidence is that a greater fraction of full-time teachersthan part-time teachers quit in their first year. The argument overlooks the possibility thatthere may be more new part-time teachers than full-time teachers. If there were, then 1/4 ofpart-time teachers may be a greater number than 1/3 of full-time teachers.

(A) and (B) raise irrelevant issues. The argument only concerns what has happened afterthe policy was introduced (“now”); what happened before doesn't matter. Remember, thatthe current attrition rates are a result of the new policy is stated as evidence; it’s not theconclusion the author is trying to establish.

(C) is an au contraire choice. If (C) were true, the argument's conclusion that more new full-time teachers quit would be correct, so (C) isn't a possibility the argument overlooks.

(E) too is an au contraire choice, since it too would make the argument's calculation correct:1/3 of a certain number is greater than 1/4 of the same number.

• It's important in critique-the-logic questions to pinpoint the conclusion, and seeexactly what is being claimed. If you understand the scope of the conclusion (in thiscase, only a comparison of numbers) you will be able quickly to reject irrelevantchoices that might otherwise seem tempting.

• Be suspicious when an argument draws a conclusion about numbers from evidenceabout proportions (or percentages). It’s an error that many of us make in everydaylife, so it’s an error that the testwriters want to see us recognize. Ask yourself: Whatquantities are the proportions based on?

15. (B)The argument concludes that the old Salmonella test should be replaced with a new test,because the new test is more accurate. But we are also told that Salmonella must be detectedquickly if treatment is to be effective. So the assumption that must be made is that the newtest provides results quickly enough to allow for effective treatment. Choice (B) restatesthis assumption, providing strong support for the argument.

(A) shifts the scope away from the heart of the argument by focusing on the skill requiredto perform the test instead of on the speed and effectiveness of the test itself.

(C) is irrelevant. The argument concerns the method of testing for Salmonella, not themethod of treating it.

(D) distorts the issue by belittling its importance to the world at large. But even if only oneperson a year gets Salmonella, the logic of the argument stays the same.

(E) The issue of remedies for Salmonella is beyond the scope. The argument is abouttesting.

Page 13: 12-PrepTest 12 Explsdl.keywin.org/5/5/559e448ca144e5a6335f12b48619009d.pdf · KAPLAN LSAT PREP LSAT RELEASED TEST XII EXPLAINED A Guide to the October, 1994 LSAT KAPLAN The answer

LSAT PREP ________________________________________________________________ LSAT Test XII Explained: Section I

© K A P L A N 11

• An argument that calls for the introduction of a new method or scheme typicallyshows how the new method surpasses the old method in a particular way. But to belogically valid, the argument must also assume that the new method is equal to orbetter than the old method in all other important ways.

• When strengthening and/or weakening an argument, consider only statementswhich affect the conclusion that is actually made. You should never be thinking:“Why is this argument true?” or “Why is this argument important?” You should ask,“Is it reasonable to reach this conclusion based on this evidence?” and “How can Istrengthen/weaken the connection between the evidence and the conclusion?”

16. (B)The argument concludes that the new Salmonella test should replace the old test becausethe new test is more accurate. However, the new test detects only “the presence or absenceof Salmonella.” If the new test can’t distinguish between Salmonella levels that are healthrisks and those that aren’t, then it really isn’t very useful. So Choice (B) does a good job ofweakening the argument.

(A) makes a useless distinction. The new test may not be able to identify other bacteria thanSalmonella, but who says that the old test can? It doesn’t affect the argument.

(C) is an irrelevant issue. The argument is about testing for Salmonella. Other intestinalillnesses are beyond the scope.

(D) It’s hard to see how associating the test with “dramatic advances in biological science”weakens the recommendation for its use. It’s useless background information.

(E) If anything, knowing that Salmonella symptoms are often misidentified underscores theneed for the accurate new test. This isn’t going to help us weaken the argument.

• Most of the wrong answer choices on a weaken the argument question will addressissues that aren’t essential to the argument. Learn to eliminate these “beyond thescope” choices quickly, and you’ll have more time to concentrate on finding theright answer.

• Don’t be intimidated by scientific language and ideas in Logical Reasoning stimuli.The LSAT hardly ever requires you to understand the actual scientific principlesinvolved. It’s the logic of the argument that’s important.

Page 14: 12-PrepTest 12 Explsdl.keywin.org/5/5/559e448ca144e5a6335f12b48619009d.pdf · KAPLAN LSAT PREP LSAT RELEASED TEST XII EXPLAINED A Guide to the October, 1994 LSAT KAPLAN The answer

LSAT PREP ________________________________________________________________ LSAT Test XII Explained: Section I

12 © K A P L A N

17. (C)The evidence consists first of a statement that, on average, members of one group (driversusing the new system) have a trait (on-time record) superior to members of another group(those on the old system). Then an agent (Millicent) is cited as #1 in that trait (best on-timerecord). Thus, so it goes, she must be a member of the first group. Well, not so: Since theargument is based on averages for each group, it’s quite possible for the driver with thebest on-time record of all to be part of either group. But substitute (C)’s “that tomato” forMillicent, “largest” for best on-time record, “experimental plot” for new system, and“regular plot” for old system, and the same (crummy) logic is at work.

(A) Very close in form to the original, incorrect because it’s not flawed logic! If in fact allpost-1988 vehicles are bigger than all pre-1988 vehicles, then it has to be true that the largestvehicle of all was post-1988, as (A) cites.

(B) would be closer to the stimulus if its conclusion, rather than its evidence, were aboutHenri’s certain membership in a group. Anyhow, (B)’s conclusion is too qualified (“tallerthan most”) to match up to those of the stimulus and choice (C).

(D),(E) The very fact that the stimulus concludes that something must be true right thenand there eliminates (D) and (E), each of whose conclusions is that something “probably”occurred.

• Start a Parallel Logic question by locating the conclusion, and tossing out choiceswhose conclusion is manifestly different. Doing that here reduces your chances to 1-in-3; sometimes the tactic gets you even further than that.

• Note that correct choice (C), here, cites the pieces of evidence in reverse order fromthe original. No matter. The evidence and conclusion in the right answer don’t haveto be stated in the same order as the original; they simply must be of the same type.

• Don’t read question stems too quickly. If you miss the reminder that the rightanswer and stimulus are flawed, you might very well be tempted by (A), whose formis awfully close.

18. (A)Erik's response simply sidesteps Frieda's reasoning, accepting the fact that lightning causesfires, but saying that other things cause a lot of fires, too. That's hardly a reason for notdoing something about the lightning damage. Erik's problem is that he gives absolutely noreason for not following Frieda's recommendation and putting up lightning rods.

(B) It's true that Erik fails to offer any additional way of combatting lightning damage, butthat's not the point. Even if he did have other ideas, that wouldn't indicate that Frieda'srecommendation also shouldn't be acted on.

(C) is absurd; there’s no appeal to emotions in Erik's talk about circuits and wiring. Theuse of emotional language — if that’s what (C) implies Erik is doing; he sounds rathermatter-of-fact to us — does not in and of itself constitute an “appeal to emotion.”

Page 15: 12-PrepTest 12 Explsdl.keywin.org/5/5/559e448ca144e5a6335f12b48619009d.pdf · KAPLAN LSAT PREP LSAT RELEASED TEST XII EXPLAINED A Guide to the October, 1994 LSAT KAPLAN The answer

LSAT PREP ________________________________________________________________ LSAT Test XII Explained: Section I

© K A P L A N 13

(D), speaking of “irrelevant,” draws an irrelevant distinction. Erik says overloaded circuitsand faulty wiring cause more fire damage than lightning, but he never compares these twofire hazards to one another.

(E) Erik doesn’t bring up the notion of causing inconvenience. His concern throughout hisargument is only with preventing damage. (In fact, it's hard to see how anyone, even Erik,could confuse “causing inconvenience” with “preventing damage.”)

• Reject any choices, like (C) and (E), that don't appear to make any sense.

• Read the choices for meaning: Don't get put off by a choice only because it doesn'tput things exactly as you would put them. If you pre-phrased the answer above(which was right and proper to do), you probably didn't think in terms of “offsettingdisadvantages.” Nevertheless, you should have been able to recognize that (A) is justa fancier way of communicating “he gives no reason not to do what Frieda said.”

19. (B)The argument depends on a scope shift: the conclusion is based on a comparison of thechange over 8 years of the automobile fatality statistics for children aged 4 and under with achange over the same period in automobile accident statistics for the entire population. Inorder to make the argument work logically, we must assume that this scope shift is valid:that the proportion of car accidents involving young children has stayed the same in thepast 8 years.

(A) Children aged 5 and up are outside the scope of the argument, whether they wearchild safety seats or not.

(C) doesn’t affect the argument. How much time children spend in cars doesn’t directlyaffect how often they get into serious accidents or how often they are killed in thoseaccidents. What if parents have learned to drive more carefully over the last 8 years?

(D) makes a useless distinction. Since the statistics which support the argument don’tdistinguish between children aged 2 and under and children aged 3 or 4, any statementbased on such a distinction is irrelevant.

(E) weakens the argument slightly. Since the number of auto fatalities for adults rose at thesame rate as the number of auto fatalities for children 4 and under, and since the adultsobviously weren’t wearing child safety seats, we’d have to question whether it’s the seatsthat deserve the credit.

• This is a great question for using the strategy of elimination. The logical justificationfor the correct answer is there, but it’s a little murky, especially if you’re notcomfortable with statistics. But three of the wrong choices can be eliminated as beingbeyond the scope, and the fourth weakens the argument.

Page 16: 12-PrepTest 12 Explsdl.keywin.org/5/5/559e448ca144e5a6335f12b48619009d.pdf · KAPLAN LSAT PREP LSAT RELEASED TEST XII EXPLAINED A Guide to the October, 1994 LSAT KAPLAN The answer

LSAT PREP ________________________________________________________________ LSAT Test XII Explained: Section I

14 © K A P L A N

20. (D)An uncharacteristically loopy stimulus from the staid LSAT writers. The argument claimsthat Joan and her friends must have defective senses of smell because they don't likeAurora's smell as much as that of other perfumes, whereas perfume expert Jameson likesAurora's smell more than that of any other perfume. This is absurd; Joan and her friendsare expressing personal preferences, and these preferences can't be overruled by anyone.There's no reason that an expert in smell physiology (from this she makes a living?) shouldbe accepted as the last word on which perfume smells best. The appeal to expert authorityis clearly illegitimate.

(A) The argument never calls the truthfulness of Joan and her friends into question, onlytheir senses.

(B) tackles an irrelevant issue. It doesn't matter how much Joan and her friends actuallylike these perfumes, the problem is that they like Aurora's smell least, and the expert likesit most.

(C) distorts the flaw. Nothing would change if more experts were called upon; the problemis that expert testimony is inappropriate in this case.

(E) The only position that the reasoning is directed against is the preference of Joan andher friends for perfumes other than Aurora, and that’s not misrepresented.

• This question and the one that follows it are clear evidence that the LSAT is notaverse to throwing in some very manageable questions late in an LR section — nooffense if you thought that one or both of them smelled.

21. (C)It's virtually impossible to pre-phrase an inference here; the only course is to look throughthe choices till you find something that strikes you as deducible. We're told that none ofJoan's friends likes Aurora's smell as much as the smell of other perfumes. We're also toldthat, by contrast, Jameson does like the smell of Aurora more than that of any otherperfume. Plainly, Professor Jameson is not one of Joan's friends. This is something of aformal logic question in disguise.

(A) Although the stimulus fails to identify any of Joan's friends as an expert on the sense ofsmell, that’s not sufficient reason to conclude that none of her friends is such an expert(though assuredly not an expert of Professor Jameson's stature in the aroma world).

(B) Joan is said to prefer the smell of all comparably priced perfume to Aurora's smell; theremay be a perfume in another price range that she likes even less.

(D) subtly changes the terms of the argument. Joan's friends don't like the smell of Auroraas much as the smell of other perfumes, but that doesn't mean that none of them likesAurora at all. Maybe some of them like all perfumes.

Page 17: 12-PrepTest 12 Explsdl.keywin.org/5/5/559e448ca144e5a6335f12b48619009d.pdf · KAPLAN LSAT PREP LSAT RELEASED TEST XII EXPLAINED A Guide to the October, 1994 LSAT KAPLAN The answer

LSAT PREP ________________________________________________________________ LSAT Test XII Explained: Section I

© K A P L A N 15

(E) goes beyond the scope. How Joan and her friends feel about other kinds of perfumes isan irrelevant issue. The only issue that the stimulus deals with is how they feel aboutAurora's smell compared to other perfumes' smells.

• If you’re aware of small changes in wording that alter the terms of the discussion(e.g. “other perfumes” vs “other comparably priced perfumes”) you can quicklydiscard improper inferences.

• When you have trouble prephrasing, don't try to force it; go immediately to theanswer choices, where the proper inference is waiting.

22. (C)The evidence is that the number of award-winning salespeople has gone down over thepast 15 years. The president concludes that since the criterion for winning is being in thetop third of the sales force, the number of salespeople who haven’t received awards — i.e.,the remaining two-thirds of salespeople — has also gone down. This only makes sense ifthe criterion for winning hasn’t changed. If, say, up until 10 years ago salespeople gotawards for sales exceeding a particular dollar amount, then the president’s conclusionwouldn’t follow.

(A) Scope shift. The issue isn’t criteria for hiring, but criteria for being a top salesperson. Itwouldn’t matter if Wilson’s had been lowering its hiring standards as long as the standardsfor receiving awards hadn’t changed.

(B) As Jean-Paul Sartre would say, “Au contraire!” The president is implying that the totalnumber of salespeople has declined, hence the decline in both award-winning and run-of-the-mill salespeople. Since (B) would weaken this argument, it can’t be assumed.

(D) Since total sales aren’t part of the argument, this choice is irrelevant. Denying (D) andassuming that average total sales figures have been increasing wouldn’t change thepresident’s conclusion a whit.

(E) It isn’t necessary that sales figures be calculated in the same way (what exactly does thatmean, anyway?), just that the criteria for winning haven’t changed. And (E) misses thepoint even further by talking only about 15 years ago, thereby avoiding the interim periodthat’s the crux of the matter.

• Use the Denial Test to check answers to assumption questions. If the negation of ananswer choice makes the argument fall apart, that choice is assumed. If the argumentis unscathed, that choice is wrong.

Page 18: 12-PrepTest 12 Explsdl.keywin.org/5/5/559e448ca144e5a6335f12b48619009d.pdf · KAPLAN LSAT PREP LSAT RELEASED TEST XII EXPLAINED A Guide to the October, 1994 LSAT KAPLAN The answer

LSAT PREP ________________________________________________________________ LSAT Test XII Explained: Section I

16 © K A P L A N

23. (C)The conclusion is about “many occurrences that shouldn’t have taken place” — and thatalone might have made you suspect that the right answer was (C), the only choice whoseconclusion is precisely in that form. Upon investigation, we see that both the stimulus and(C) say that a course of action (capturing wild creatures/punishing little kids) has only onejustification (saving life/future welfare), but that many times, that course of action is beingcarried out for another reason altogether (breeding in captivity/minimizing siblingrivalry).

(A) and (E) each feature a conclusion infinitely more categorical than that of the stimulus;neither concludes with the assertion that “many” events should not have occurred.

(B) lacks the “something is justified only if . . .” element contained in both the stimulus andcorrect choice (C). Instead, the first sentence deals with a group (parents who don’t punishtheir children) that’s not justified in doing something (complaining), and simply goes on tosay they do it anyway. Lacking this parallel form of a justification, (B) naturally lacks the“alternative to the justification” element contained in the second sentence of the stimulus(only justification: saving animal’s lives; alternative: breeding in captivity).

(D) also lacks the “alternative justification” element of the stimulus. Indeed, (D) lacksjustification altogether: The explicit permission clause is cited as a necessary preconditionfor punishment, not a justification for same.

24. (A)The author first concludes that the presence of titanium in the ink of the Gutenberg and B-36 Bibles supports the theory that Gutenberg printed both. Okay, this hinges on titanium’sbeing a rare ingredient in 15th-century ink and therefore characteristic of Gutenberg. Thenthe author concludes that the presence of titanium in the ink of the Vinland Map suggeststhat the map dates from the 15th century. But that conclusion hinges on titanium’s beingtypical of 15th-century ink. Clearly, both conclusions can’t be true: If titanium was typicalof 15th-century ink, then the B-36 Bible could have been printed by anyone.

(B) and (D) contain classic scope shifts. In (B), the issue of whether 15th-century printersand artists knew titanium was in their ink was never raised by the author. As for (D), theauthor never implies the B-36 Bible and the Vinland Map can’t be appreciated on their ownmerits, so this can’t be a problem in his reasoning.

(C) Read carefully! Although the author does use titanium’s presence to determine —unreasonably — when the Vinland Map was printed or drawn, he doesn’t use it todetermine where it was printed or drawn. More importantly, (C) misses the author’s majorflaw, that of drawing inconsistent conclusions.

(E) introduces an irrelevant distinction. The author never says exactly when thesediscoveries occurred, just that they happened “recently.” The problem isn’t whichdiscovery occurred first; it’s that the author uses them to draw conflicting conclusions.

Page 19: 12-PrepTest 12 Explsdl.keywin.org/5/5/559e448ca144e5a6335f12b48619009d.pdf · KAPLAN LSAT PREP LSAT RELEASED TEST XII EXPLAINED A Guide to the October, 1994 LSAT KAPLAN The answer

LSAT PREP ________________________________________________________________ LSAT Test XII Explained: Section I

© K A P L A N 17

• Reading the question stem first, while helpful with all Logical Reasoning questions,is an especially handy strategy for flaw questions. You save time by knowing at theoutset that there’s something wrong with the reasoning in the stimulus.

• As you go through the answer choices for flaw questions, ask yourself whetherfixing the “flaw” listed in each choice will repair the argument. If there’s still aproblem with the reasoning, look for a different choice.

• Pay meticulous attention to detail when your read choices. The reference to“location” in (C), for example, is enough to rule out this choice.

25. (B)A pure formal logic question: quite rare on today’s LSAT. The key, as so often, lies in thequestion stem: Four of the choices are definitely true, so we are to find the choice that neednot be true.

One approach is to go in search of a choice to which you can find an exception. The firsttwo statements act as a simple syllogism: If all actors are exuberant, and all exuberantpeople are extroverts, then we may conclude that all actors are extroverts. Fine. The shypeople who, according to the last part of the sentence, are actors, must be extroverts (likeall actors), and indeed all shy extroverts could be actors. Contrary to (B), there could be nonon-acting shy extroverts.

Find that troublesome? Perhaps it’s easier to confirm one or more choices as definitelytrue, and proceed from there.

(A),(E) Since all actors, as we showed above, are extroverts, then some shy people — theones who are actors, at least — must be extroverts. “Some” statements, you’ll recall, arereversible: “Some X are Y” = “Some Y are X,” so if some shy people are extroverts, thensome extroverts are shy.

(C) All actors are exuberant, says the stimulus, and some shy folks are actors. They,therefore, are people who are shy and actors and exuberant, as (C) says.

(D) must be true because it’s the contrapositive of the conclusion that appears in boldfaceabove. “All actors are extroverts” = “If you’re an actor, then you’re an extrovert” = “Ifyou’re not an extrovert, then you’re not an actor” = (D).

• This type of question doesn’t appear in LR often enough to worry about. Rememberthe techniques, practice them when you can, and don’t be unwilling to skip past aformal logic question if you hate them.

Page 20: 12-PrepTest 12 Explsdl.keywin.org/5/5/559e448ca144e5a6335f12b48619009d.pdf · KAPLAN LSAT PREP LSAT RELEASED TEST XII EXPLAINED A Guide to the October, 1994 LSAT KAPLAN The answer

LSAT PREP ________________________________________________________________ LSAT Test XII Explained: Section I

18 © K A P L A N

26. (E)The claim derived from the study is simply that “some well-managed farms” havesuccessfully employed natural farming methods — in other words, that these methods canbe used with success. Since the conclusion isn’t that natural methods are generally moresuccessful, the critics’ objection that the study omitted unsuccessful farms is irrelevant.

(A) The critics don’t argue that successful farming is merely a matter of luck, so this isn’t arelevant criticism of their response.

(B), like the critics, misinterprets the study’s results. Since the only result claimed is thatsome — not most — farms successfully used natural methods, including twice as manyfarms won’t change this.

(C) Irrelevant! The critics don’t address the reasons for the methods’ success or failure, sothey make no assumptions about the role of soil quality.

(D) Be careful! The critics object — irrelevantly — that natural methods aren’t necessarilysuitable for all farmers, but they don’t demonstrate that this is indeed true. To do so, theywould have to present new evidence in their response. Although they criticize the study’sevidence, they don’t present any new evidence of their own.

• In dialogues, be sure to isolate the argument each person is making. Read the secondargument with respect to its relation to the first.

• Ordinarily, in dialogue-format questions, it’s the first speaker who makes the logicalerror and the second speaker who makes a successful rebuttal. Here’s an exceptionwhere it’s the responding comment that blows.

Page 21: 12-PrepTest 12 Explsdl.keywin.org/5/5/559e448ca144e5a6335f12b48619009d.pdf · KAPLAN LSAT PREP LSAT RELEASED TEST XII EXPLAINED A Guide to the October, 1994 LSAT KAPLAN The answer

© K A P L A N 19

SECTION II:

LOGIC GAMES

Page 22: 12-PrepTest 12 Explsdl.keywin.org/5/5/559e448ca144e5a6335f12b48619009d.pdf · KAPLAN LSAT PREP LSAT RELEASED TEST XII EXPLAINED A Guide to the October, 1994 LSAT KAPLAN The answer

LSAT PREP _______________________________________________________________ LSAT Test XII Explained: Section II

20 © K A P L A N

GAME 1 — Piano Lessons(Q. 1-6)

The Action: A fairly straightforward sequencing game. We’re asked to schedule six peoplefor piano lessons, one per day for six days. The Key Issues in this game are about:

1) On which day is each person’s lesson scheduled?2) Who comes before and after whom in the schedule?

The Initial Setup: The setup for such a straightforward sequencing game is pretty basic;use dashes or numbers and list the cast of characters off to the side:

1 2 3 4 5 6 G H J S T U

The Rules:

1) “J . . . H” is one good way to represent this rule visually, such that you can quicklyrecall and access it. Learn from your mistake if you absent-mindedly jotted down “H...J.”

2) By the same token, “S . . . U” will suffice for this one.

3) This means that in the schedule — and in whatever sketch we may draw — S is threespaces after G. “G __ __ S” will serve our purposes.

4) This one simply tells us that J will either be in space 1 or space 3, and remember, Rule 1tells us that wherever J ends up, she’s before H. You might wish to shorthand this byplacing your “J . . . H” from Rule 1 above your sketch, with arrows pointing from J tospaces 1 and 3.

Key Deductions: Combining Rules 2 and 3 gives us “G __ __ S . . . U.” The differencebetween the Logic Games Mediocre and the Logic Games Elite is that the former willignore this large block and move right to the questions, whereas the latter will besuspicious of a chunk that takes up that much space. Surely, something can be done withit, right? Right: This “G __ __ S . . . U” block takes up at least five of the six possiblespaces in the game, and from this we can deduce that G must either be in space 1 or space2 (if G’s any further over, U gets pushed off the schedule entirely). At Kaplan, we call this alimited option; knowing that G is limited to one of these two spots gives us a majoradvantage in every question. We can also work through the implications of each:

Page 23: 12-PrepTest 12 Explsdl.keywin.org/5/5/559e448ca144e5a6335f12b48619009d.pdf · KAPLAN LSAT PREP LSAT RELEASED TEST XII EXPLAINED A Guide to the October, 1994 LSAT KAPLAN The answer

LSAT PREP _______________________________________________________________ LSAT Test XII Explained: Section II

© K A P L A N 21

If G is in 1, then S is in space 4 (Rule 3) and U is in either 5 or 6 (Rule 2). Also, J will haveto be in space 3 (Rule 4), leaving H to float with U in 5 or 6. This leaves T in space 2. Inother words, if G is in 1, almost everything is determined, like so:

G in 1: 1 2 3 4 5 6G T J S H/U H/U

We can’t deduce as much if G is in 2, but we do know that S would be in space 5, whichplaces U definitely in space 6:

G in 2: 1 2 3 4 5 6G S U

In either case, we get a ton of information that will help us rack up easy points.

The Final Visualization: So, we have the different options above, and the info below withwhich to attack the questions:

1 2 3 4 5 6G __ __ S … U

J … H

The Big Picture:

• Groups or blocks of entities that take up a lot of space in the game often lead to“limited options” for one or more of the entities. The concept of “limited options” isextremely valuable. In this case, at the beginning of most questions, or certainlywhen you’re stuck, you can focus on trying out G on day 1 or day 2, and the limitednumber of schedules that results, many of which we’ve already worked out. Havingthis info up front is bound to lead to some quick and easy points.

• Remember, whenever you make a deduction, no matter how small, don’t stop there—it may lead to something bigger. Take the time to explore the next step. The timeyou’ll save answering the questions will more than make up for the effort.

• When a rule states that one entity is “three day after” another it means that there areexactly two spaces between them. Make sure you read the rules carefully, and eventry out an example of the rule if need be to clarify its meaning.

Page 24: 12-PrepTest 12 Explsdl.keywin.org/5/5/559e448ca144e5a6335f12b48619009d.pdf · KAPLAN LSAT PREP LSAT RELEASED TEST XII EXPLAINED A Guide to the October, 1994 LSAT KAPLAN The answer

LSAT PREP _______________________________________________________________ LSAT Test XII Explained: Section II

22 © K A P L A N

The Questions:

1. (E)We deduced above that G must be scheduled for day one or day two, so if J is on day one,then G must be on day two. Steve’s lesson must therefore be on day five (Rule 3), whichplaces Una on day six (Rule 2).

• When you’ve done excellent deductive work up front, reward yourself by trusting it.Don’t be suspicious of such an easy question, and don’t waste valuable time testingout all the choices when you’re sure about your work. Circle it, feel good aboutyourself, and move on.

2. (B)Simply put, this one asks who could be scheduled on both days three and five. Let’s playwith our “limited option” info: the insight that Grace can only be scheduled on day one ortwo. As you’ll recall, if Grace is on day one, then either Henry or Una must be on day five.If Grace is on day two, then Steve must be on day five. So the only people who canpossibly be on day five are Henry, Una, and Steve, which eliminates choices (A) and (D).Now check day three: Rule 3 eliminates Steve, choice (C); the earliest he could bescheduled is day four. By extension, Rule 2 axes Una, choice (E), since she’s scheduled evenlater than Steve. Only Henry, choice (B), could be scheduled for both day three and dayfive.

• Some games, such as the “process” game at the end of this section, require you toconstantly hark back to the individual rules. However, when you’re able to work outas many scenarios as we did in this game’s setup, use this information toaggressively pursue the right answer.

• Decode lengthy question stems. Here the testmakers use a really long and official-sounding stem to ask a pretty simple question: “Who could be scheduled for bothdays 3 and 5?”

3. (C)Here’s where the work we put in up front really pays off. We deduced that there are onlytwo options for G: day one or day two. If G is on day 1, then T is on day 2. (If you forgethow we deduced this, go back and reread the Key Deduction section.) If, on the otherhand, G is on day 2, well, then, yeah, G is on day 2 (now there’s the kind of logic thetestmakers reward!). The “limited options” that we discovered for G tell us that these arethe only two scenarios possible, so G and T are the only possibilities for day 2.

• The importance of exploring deductions and recognizing “limited options” shouldbe crystal clear by now. The information we uncovered during the setup of thisgame has turned an otherwise normal game into one that so far is yielding ten-second questions and easy points.

Page 25: 12-PrepTest 12 Explsdl.keywin.org/5/5/559e448ca144e5a6335f12b48619009d.pdf · KAPLAN LSAT PREP LSAT RELEASED TEST XII EXPLAINED A Guide to the October, 1994 LSAT KAPLAN The answer

LSAT PREP _______________________________________________________________ LSAT Test XII Explained: Section II

© K A P L A N 23

• Don’t assume that you have to “draw” something for every question. When you’vedone the right work up front, glancing at your scratch work is all that’s necessary toanswer a question like #3.

4. (B)In keeping with the pattern, this one plays right into our hands: It’s concerned with thescheduling of G’s lesson, and there ain’t nothin’ we know more about in this game than thescheduling of G’s lesson. Our discovery that Grace must be on day one or two allows us toquickly cross off (C), (D), and (E). And when Grace is on day one, we saw that Tom mustbe on day two with Janet on day three (Key Deductions), which violates the mandate in thequestion stem, and therefore effectively kills (A). All that will work is Grace on day two:J G H T S U or J G T H S U are both acceptable schedules under these circumstances.

• The Bullet Point O’Wisdom here is that there’s no bullet point necessary: Surely bynow, the value of our Key Deductions, and especially the “limited options” for G,speaks for itself.

5. (C)It’s getting pretty repetitive, this “Grace thing” leading to all the answers, but hey, when itcomes to quick and easy points, we’ll take ‘em where we can get ‘em. “J on three” — nowwhere have we seen that before? Well, in the “Grace on day one” option, for one. In thisscenario, Henry and Una float between days five and six. It’s therefore quite possible toschedule Una on day five and Henry on day six, verifying that choice (C) could be true.

We can rule out (A) and (B) because we know from our Key Deduction options that Henryand Una can never be scheduled before Grace. The only way to schedule Tom later thanHenry is if Janet’s on day one, which she’s not in this question, so (D) is out. Finally, ifGrace is on day one, Tom must be on day two with Una on day five or six. If Grace is onday two, Una must be on day six. So Tom’s lesson, contrary to (E), can’t be later than Una’s.

• Approach a “could be true” question differently from a “must be true” question. Inthe latter, as soon as you come across a scenario that “works” in one instance, yourwork is done. In this case, of course, you could have scanned the choices, comparingthem to the options that we’ve been referring back to for the whole game.

Page 26: 12-PrepTest 12 Explsdl.keywin.org/5/5/559e448ca144e5a6335f12b48619009d.pdf · KAPLAN LSAT PREP LSAT RELEASED TEST XII EXPLAINED A Guide to the October, 1994 LSAT KAPLAN The answer

LSAT PREP _______________________________________________________________ LSAT Test XII Explained: Section II

24 © K A P L A N

6. (D)Back to the options for Grace — hey, if it ain’t broke, well, you know the rest. Anyway, ifGrace is on day one, Tom must be on day two. If Grace is on day two, Steve is on day fiveand Una is on day six. There are a few possibilities to follow up on here. Tom could be onday one with Janet on day three and Henry on day four. But if Janet’s on day one, thenHenry and Tom are both possibilities for days three and four. So Tom’s pretty flexible —he can take his lesson on days one, two, three, or four.

• Here’s a game in which every single question was helped along by the realizationthat G is faced with very limited options, and that there are only a few possible waysto play out the situation in each. If you didn’t see this up front, don’t despair: Thatdoesn’t mean that you couldn’t do well on this game. Even if you didn’t get far withdeductions up front, you should have begun to notice patterns emerging as youprogressed through the questions. In the best of cases, we’re able to pick up on lotsof great stuff in the setup stage, so it pays to get into the habit of making the most ofSteps 1 - 4 in the Kaplan Approach.

Page 27: 12-PrepTest 12 Explsdl.keywin.org/5/5/559e448ca144e5a6335f12b48619009d.pdf · KAPLAN LSAT PREP LSAT RELEASED TEST XII EXPLAINED A Guide to the October, 1994 LSAT KAPLAN The answer

LSAT PREP _______________________________________________________________ LSAT Test XII Explained: Section II

© K A P L A N 25

GAME 2 — Canoe Trip(Q. 7-11)

The Action: Couldn’t be any more explicit — the whole game centers around breaking uppeople into three groups. Specifically, we’re asked in this grouping game to distribute ninepeople (children F,G,H,J, and K, and adults Q,R,S and T) into three groups for a canoeingtrip, three per group. The Key Issues are pretty much what we’d expect from this familiargame type, namely:

1) Who’s in what group?2) Which people can or must be in the same group?3) Which people can’t be in the same group?

The Initial Setup: The setup is fairly simple — use the numbers 1, 2, and 3 for the groupsand place exactly three dashes under each, as specified in the intro. Off to the side, jotdown the entities broken up by category (children and adults). You may wish to usecapital and lower-case letters to distinguish the adults from the children, if you feel thishelps keep the entities straight; for our purposes here, and to show you that it can be done,we’ll use all caps.

1 2 3__ __ __ __ __ __ __ __ __

CFGHJK

AQRST

The Rules:

1) There must be at least one adult per group. Well, there are three groups, and fouradults, so do the math. If you mentally distribute one adult into each group, that leavesone extra adult. When we add that one in, we get two groups with one adult and one groupwith two adults, so remember it and maybe jot that down: adults 1, 1, 2. (Of course, wedon’t know which group gets the double dose of adults.)

2) F and J are together in the same group; let’s write it as F = J or “ALWAYS FJ.”

3) G and T are not in the same group: G≠T or “NEVER GT.”

4) H and R are not in the same group: H≠R or “NEVER HR.”

5) “No H or T” over the “2” in our sketch will do for this one. We ought to take a momentto realize that that means that both H and T must be placed in group 1 or 3 (though notnecessarily together).

Page 28: 12-PrepTest 12 Explsdl.keywin.org/5/5/559e448ca144e5a6335f12b48619009d.pdf · KAPLAN LSAT PREP LSAT RELEASED TEST XII EXPLAINED A Guide to the October, 1994 LSAT KAPLAN The answer

LSAT PREP _______________________________________________________________ LSAT Test XII Explained: Section II

26 © K A P L A N

Key Deductions: Notice that three of the five rules are stated in the negative: who’s notwith whom, and who’s not in a particular group. It’s much harder to formulate deductionsfrom these types of rules. Instead, we’ll have to use the rules directly to answer many of thequestions.

However, there are a few implications of Rules 1 and 2 worth noting: We saw from Rule 1that the adults must be split up like so: 1, 1, 2. Taking this one step further, we can deducethat the children will be split up into groups of 2, 2, and 1 to add up to 3 in each group.This isn’t an earth-shattering deduction, but it may prove somewhat helpful along the way.The other thing to notice comes from Rule 2: If F and J, two children, must be together,then the third member of that group must be an adult. Therefore, F and J can’t be groupedwith any of the other children; namely, G, H, or K.

The Final Visualization: Here’s what we’re armed with as we move on to the questions:

1 2 3__ __ __ __ __ __ __ __ __

CFGHJK

AQRST

F = JG ≠ TH ≠ R

No H or T

The Big Picture:

• When you’re given numerical information (such as “at least one adult per group”),work out the simple arithmetic — doing so will usually help you to further clarifythe placement of entities when you get to the questions.

• Learn to determine which rules will help you the most. While it’s important to keeptrack of who’s not together, that’s not as concrete and therefore not as valuable asknowing who is together. In this game, the FJ group is a constant — in manyquestions, you’ll be best served by focusing on their placement first.

• To see whether or not a game is ripe for making deductions, look for rules thatcontain common entities. The people in Rules 2, 3, and 4 don’t repeat (F and J, G andT, H and R), so it’s no surprise that we can’t combine these rules to form a majordeduction. (Rules 4 and 5 have H in common, but combining these rules doesn’thelp us much either.)

• A quick scan of the question stems can tell you a lot about a game. Here, thequestion stems all begin with the word “if,” which is another clue that there may notbe a major deduction lurking. In a subtle way, the testmakers are telling us that theyhave to give us more information in order for something meaningful to happen ineach question (well, as meaningful as things get on the LSAT).

• Conversely, if you scan through the question stems up front and find one that asks:“Which one of the following must be true?,” then you know immediately that there’sat least one thing deducible from the intro and the rules alone, and you should lookfor it when you set the game up.

Page 29: 12-PrepTest 12 Explsdl.keywin.org/5/5/559e448ca144e5a6335f12b48619009d.pdf · KAPLAN LSAT PREP LSAT RELEASED TEST XII EXPLAINED A Guide to the October, 1994 LSAT KAPLAN The answer

LSAT PREP _______________________________________________________________ LSAT Test XII Explained: Section II

© K A P L A N 27

The Questions:

7. (E)Where F goes, J goes (Rule 2), which means that group 1 already has its fill of children. G,H, and K must therefore go into group 2 or 3. Since H is barred from group 2 (Rule 5), Hmust be in 3. Nothing else is absolutely apparent, so let’s go to the question, which asks usto find a statement that could be true. Well, we need to simply check each choice againstthis scenario (F/J in 1, H in 3):

(A) Impossible: GK and H in group 3 gives us a full kiddie boat, which may very wellcapsize without an adult aboard (Rule 1 — alright, you know from Logical Reasoning that“capsizing” is outside the scope here, but we threw that in for dramatic effect).

(B) Nope; H is in 3, and R can’t join H anywhere (Rule 4).

(C) No good; as we just saw, J must join F in 1.

(D) F and J are already in group 1, so K and R joining them would give us four people inthat group, which violates the intro’s mandate of three per group.

(E) This choice must be good, since all the others stink. Just for the record, FJR / QSG /HKT is one acceptable distribution for groups 1, 2, and 3 respectively, that’s in sync withchoice (E).

• This “just for the record” stuff is something we at KAPLAN have to include here,just in case one of you can’t see how (E) is possible. On test day, don’t waste valuabletime filling in the groups just to make sure. Logic Games answers are objectivelycorrect, so after eliminating (A) through (D), have the confidence to know that (E) hasto be the one.

• Use the first question in a game to solidify your conception of the situation. Youshould get quickly acclimated to the idea that wherever F and J go, children G, H,and K must go somewhere else, and their choice is limited to the remaining twogroups.

8. (A)This time we’re given the adult that joins F, and by extension, J, so put S, F and J into group3. Now that group 3 is full, we’ll have to put H and T into group 1, thanks to Rule 5. Eitherone of those necessities among the answer choices? Nope, so we’ll have to press on. Rules 3and 4 help a lot — G and T must be split up, and H and R must be split up, so since H andT are in 1 and group 3 is full, we can place G and R into group 2. Either of those pieces ofinformation among the choices? Yup; G in 2, choice (A).

As for the remaining choices, we’ve seen how (B) and (E) are downright false, while (C) and(D) are both possible, but not necessary, as K could be in 2 and Q in 1.

Page 30: 12-PrepTest 12 Explsdl.keywin.org/5/5/559e448ca144e5a6335f12b48619009d.pdf · KAPLAN LSAT PREP LSAT RELEASED TEST XII EXPLAINED A Guide to the October, 1994 LSAT KAPLAN The answer

LSAT PREP _______________________________________________________________ LSAT Test XII Explained: Section II

28 © K A P L A N

• Use your work on previous questions to help you work through later ones, whenyou can. Here, you should think to yourself: “Placing F again? We just did that — Jmust go with F in 3, and H can’t go there or group 2, so H must be in 1....” Presto,you’re on your way.

• Any time you figure out something new in a question, it pays to take a quick look atthe answer choices. Sometimes you’ll be rewarded with a quick and easy point. Ifyour deduction isn’t there, that’s a clear sign that more steps are necessary to get youto the correct choice.

9. (E)We can get this one fairly quickly by using Rules 3 and 5. We’re told that G and K are ingroup 3, and according to Rule 3, T can’t be there if G is. But T can’t be in group 2 either,(Rule 5), so T must be in group 1.

As for the wrong choices, (A) is off because H must be in group 1 (3 already has 2 kids, andRule 5 prevents H from joining group 2). And if H is in group 1, then F and J must be ingroup 2, which kills (B). Once again, we’re left with two choices — (C) and (D) — that canbe true, but need not be.

• Don’t be thrown by a little ambiguity. With nine entities, it’s rare that we’ll actuallybe able to slot every single one into a definite space. Just strive to place as manypeople as you can, and when you’ve reached the limit (or even before, as the casemay be), the correct answer will be waiting.

10. (D)Q and S are both adults, so splitting them up in this way leaves group 2 in need of an adult(Rule 1). Adults R and T remain to be placed, but as usual, T can’t go in 2, so pencil R intothat group. We can’t find a definite spot for anyone else, so we’ll have to move on to thequestion. It will be helpful, however, before we do so, to take inventory of who’s left: theF/J pair, of course; H and T, who will be blown to bits if they get anywhere near group 2(oops, there’s that dramatic license again, but hey, anything that helps you to remember therules is fine); and children G and K.

We’re looking for something that cannot be true, so the test for each choice is whether it canbe made to work; if it can, then we can cross it off.

(A), (B), (C) Can G be in group 2? Earlier we said that when you don’t know what the nextmove is, focus on the F/J pair. Throw ‘em somewhere, and see what happens. Of course,don’t put them in group 2, because we’re testing to see whether G can go in 2. So try F/J in3 with S. That forces H and T into 1 with Q (Rule 2), and G and K into 2 with R. Soundokay? You bet — so not only is (A) possible, but having Q, H, and T in group 1 illustratesthat (B) and (C) are possible as well. Notice how this single example allows us to cross allthree of these choices off.

Page 31: 12-PrepTest 12 Explsdl.keywin.org/5/5/559e448ca144e5a6335f12b48619009d.pdf · KAPLAN LSAT PREP LSAT RELEASED TEST XII EXPLAINED A Guide to the October, 1994 LSAT KAPLAN The answer

LSAT PREP _______________________________________________________________ LSAT Test XII Explained: Section II

© K A P L A N 29

(D) Can there be exactly one child in group 2? H and T can’t go there, which leaves onlythe children pair F/J, and children G and K. If the F/J pair doesn’t take those spots in 2,then G and K must, so no matter what, group 2 will have 2 children, which means that (D)contains the impossibility we seek.

(E) If the F/J pair go to group 1, then H and T will join S in 3, resulting in only one child inthat group.

NOTE: There is a quick intuitive way to solve this one that some of you may have noticed:With Q in 1, R in 2, and S in 3, and the fact that T can’t be in group 2, group 2 must haveonly one adult, and therefore have two children. So we can tell right away that choice (D) isimpossible.

• This may have been the hardest and/or most time consuming question of the bunch,because we were able to place only 3 entities, leaving the rest up in the air. That’s notso tough by itself, but some of the answer choices deal with the number of childrenin each group, which introduces a new level of abstraction. If you find that you’retotally baffled by the ambiguity in a question, skip it and come back to it if timepermits (especially in this case, where the next question is a nice solid “must betrue.”)

• After placing as many entities as you can, always ask yourself: “Who’s left?”

• In games like this one, in which the entities are broken into groups, ask “whatgroups do they belong to” (children or adults), and “what else do I know about themfrom the rules?”

• In a “cannot be true,” “must be false,” or “which is impossible” question, you mayneed to simply work from the choices. And all you need is one valid example thatshows that a choice can be true, in order to eliminate that choice.

11. (B)The 2/2/1 children breakdown that we deduced earlier comes in handy here. If G is theonly child in group 1, then F and J are together, as always, which forces the other kids, Hand K, together into a group. And since H can never be in group 2, and group 1 is spokenfor in the question stem, we can throw H and K into group 3, and F and J therefore into 2.Any of this good stuff represented in the choices? Well how about that, we catch a break —K in 3, choice (B). Sort of looks like a peace offering from the testmakers after that previous,somewhat annoying question.

(A) is simply wrong; F is in 2. Meanwhile, the acceptable arrangement GQS / FJR / HKT ingroups 1 - 3 respectively shows that remaining choices (C), (D), and (E) need not be true.

• Don’t be surprised to find a fairly easy question at the end of a game. After workingthrough the previous four questions, you should have a great grasp of the action andthe rules governing this game. See to it that you take at least a look at every questionin every game. Doing so may lead you to skip a more difficult one in order to spendtime on a straightforward “must be true” question like this. That would be great!

Page 32: 12-PrepTest 12 Explsdl.keywin.org/5/5/559e448ca144e5a6335f12b48619009d.pdf · KAPLAN LSAT PREP LSAT RELEASED TEST XII EXPLAINED A Guide to the October, 1994 LSAT KAPLAN The answer

LSAT PREP _______________________________________________________________ LSAT Test XII Explained: Section II

30 © K A P L A N

Game Three — Street Vendor Food(Q. 12-17)

The Action: A standard one-to-one matching game. We’re asked to match people (Lara,Mendel, and Nastassia, abbreviated of course as L, M, and N) with the kind of food orfoods they buy — fruit cups, hot dogs, pretzels, and shish kebabs. (We believe that thislunch is known as the “Healthy Heart Special.”) The Key Issues are:

1) Who buys what?2) Who doesn’t buy what?3) Which people can or must buy the same foods?4) Which people can’t buy the same foods?5) How many of each type of food are, or can be, purchased?

The Initial Setup: Either a grid or a simple list works well here; use whichever one you’remost comfortable with. For our purposes, we’ll use a 3x4 grid, or table, to keep track of theinformation, with the people (L, M, N) across the top and the foods down the side, like so:

L M N

FCHD

PSK

A check mark in a box means that a person buys that particular kind of food; an X willsignify that a person doesn’t buy that particular kind of food.

The Rules:

1) Don’t misinterpret Rule 1; it doesn’t say that a person can’t buy more than one kind offood. The game hinges on people buying at least one kind of food, which implies that theycan buy more than one kind. What the rule actually says is that nobody buys more than oneportion of each kind of food. It’s a loophole closer; it means that we needn’t worry aboutsomebody buying two hot dogs, or three pretzels, for example.

2) No one buys both a hot dog and a shish kebab. You could write that as “H ≠ S,” or areminder: “NEVER HS!!”

Page 33: 12-PrepTest 12 Explsdl.keywin.org/5/5/559e448ca144e5a6335f12b48619009d.pdf · KAPLAN LSAT PREP LSAT RELEASED TEST XII EXPLAINED A Guide to the October, 1994 LSAT KAPLAN The answer

LSAT PREP _______________________________________________________________ LSAT Test XII Explained: Section II

© K A P L A N 31

3) Up to now we’ve been wondering how many of each type are purchased, and do wehave to use all four types? This brings us closer: At least one person buys a hot dog, and atleast one buys a pretzel. Any way you choose to jot this down in order to remember this isfine; “at least 1 H and P” would suffice, or somewhere in the hot dog and pretzel row inyour grid, write “at least 1.”

4), 5) M buys a shish kebab, and N buys a fruit cup. These are the most concrete rules ofall, so go ahead and build this information directly into your sketch using big check marksin the appropriate boxes. Also notice that all four food types are accounted for: All four arepurchased, by one or more people.

6) Love these rules that don’t require too much thinking. A big X in the pretzel box for Land N captures the gist of this rule. Those not using a grid, but keeping lists, would wantto note “NO P” under Lara and Nastassia’s columns.

7) M and N have no food purchases in common. Jot down something to ensure that younever forget this requirement, and we’re ready to try to piece this thing together.

Key Deductions: This game is truly an exercise in deduction.

Rule 3 tells us that at least one person buys a pretzel, but Rule 6 informs us that it’s neitherL nor N, which means that M must buy a pretzel. And speaking of M, M buys a shishkebab (Rule 4), which means that M doesn’t buy a hot dog (Rule 2). And just when youthought we were finished with M, we find that we can also combine Rules 5 and 7 todeduce that M doesn’t buy a fruit cup. So we haven’t even gotten to the questions yet andMendel’s menu is already complete — a pretzel and a shish kebab, but no fruit cup or hotdog. Not bad. And as if that’s not enough, Mendel’s going to help us out even more: SinceM buys a shish kebab and N won’t buy any kind of food that M does (Rule 7), N won’t buya shish kebab.

The Final Visualization: Here’s what we have going for us heading into the questions:

L M N M ≠ NNever HS

✗ at least 1

✗ at least 1

✔✗

FCHD

PSK

Page 34: 12-PrepTest 12 Explsdl.keywin.org/5/5/559e448ca144e5a6335f12b48619009d.pdf · KAPLAN LSAT PREP LSAT RELEASED TEST XII EXPLAINED A Guide to the October, 1994 LSAT KAPLAN The answer

LSAT PREP _______________________________________________________________ LSAT Test XII Explained: Section II

32 © K A P L A N

With the four definites given to us by the rules, and the four things we deduced from therules, eight out of the 12 possible outcomes are already determined. We’d say we’re readyto pick up a few points, wouldn’t you?

The Big Picture:

• Don’t be intimidated by the sheer number of rules. The more information you’regiven, the less abstract the game is, and the easier it is to handle.

• Most deductions aren’t hard to make — you simply have to get into the habit oflooking for them.

• Make sure you’re in line with number terminology on the LSAT: “At least” is muchdifferent from “only” and “exactly.” Read the game setups and the rules critically!Misinterpreting one of these number-related phrases can easily ruin a game.

• Always try to turn negative rules around to the positive. When you’re told what can’thappen (L and N can’t buy a pretzel), see if you can deduce what must happen (Mmust buy a pretzel). Positive information is much more concrete and powerful thannegative information.

The Questions:

12. (D)It’s no surprise that the first question simply asks what must be true; hey, we’ve seen fourpossible answers to this question already. Since we were able to deduce Mendel’s entiremenu (see Key Deductions above), we should quickly scan the choices looking for him.(D)’s the one — Mendel buys a pretzel as a result of combining Rules 3 and 6.

(A), (E) L could buy a hot dog while N doesn’t, or vice versa. Therefore, (A) and (E) bothcould be true, but need not be.

(B) Maybe, maybe not.

(C) Mendel can’t buy a hot dog, according to Rules 2 and 4.

• When you see a “must be true” question without a hypothetical “if” statementattached, this is a big red flag that there are some deductions possible from thesetup.

• Pay particular attention to the wording of the questions. If you fail to distinguishbetween “can” and “must,” you’re in for a long day.

• Much like in Game 1 on this section, you must confidently use your deductions.That is, you don’t have to “test out” every single answer choice; rather, aggressivelypursue the answer by scanning the list of choices whenever possible.

Page 35: 12-PrepTest 12 Explsdl.keywin.org/5/5/559e448ca144e5a6335f12b48619009d.pdf · KAPLAN LSAT PREP LSAT RELEASED TEST XII EXPLAINED A Guide to the October, 1994 LSAT KAPLAN The answer

LSAT PREP _______________________________________________________________ LSAT Test XII Explained: Section II

© K A P L A N 33

13. (B)We were told that Mendel buys a shish kebab and that Nastassia buys a fruit cup. Then wededuced that Mendel buys a pretzel. And now we know the purpose of Rule 1 — the gamespecifies that no one buys more than one portion of a kind of food so that when the questionstem says that the vendor charges $1 for each portion, we can calculate that Mendel’s oneshish kebab and one pretzel comes to $2, and that Nastassia’s one fruit cup totals $1. So farwe have three food items at one dollar each to total $3, and we’re looking for the minimumamount the three could spend. Can we stop there at 3? Nope; the intro says that eachperson buys at least one food item, and Lara hasn’t bought anything yet. Furthermore, westill have to satisfy Rule 3 — at least one person must buy a hot dog. We can take care ofboth in one fell swoop by having Lara buy a hot dog. That’s one more item at one moredollar, giving us a minimum of $4 spent.

• Remember Grace from Game 1 on this section? Well, Mendel’s our Grace in thisgame — the entity about whom we know so much that it pays to focus on him inpractically every question.

• Logic Games rarely involve math. However, occasionally some simple number-related questions will show up on the test. Don’t let them throw you; you’ll neverhave to do anything more complex than simple addition or subtraction. This sectionexists to test logic and quick deductive thinking, not your math skills.

14. (B)This question is very similar to the previous one, except this time we’re asked for themaximum amount that the three people can spend. Mendel is still fixed with his two itemsfor $2. Nastassia definitely buys a fruit cup, and she can buy a hot dog, so let’s throw thatin. She can’t buy a pretzel (Rule 6), and we’ve deduced that she can’t buy a shish kebab, soNastassia is also finished, with two items for $2. The only thing we know about Lara is thatshe doesn’t buy a pretzel, so she’s left with three food possibilities. Surely she can buy afruit cup easily enough, but Rule 2 forces her to choose between a hot dog and a shishkebab. Whichever one Lara chooses, her maximum will also be two items for $2, whichbrings us to a total of six items for $6.

• In number-related questions, you generally make the best use of time by figuringout the correct answer and then finding it among the answer choices, rather thanlaboriously testing out the various choices. And any time saved in typical games,such as the first three on this section, will probably be sorely needed to tackle gameslike “Chemicals in Flasks” at the end of this section.

• Pick up on common elements used in different questions. If you took the time tothink through this $1 per portion thing from Question 13, you don’t have to rethinkit now. After reading the stem, you should have thought: “Oh, just the opposite ofthe last one. Simple.”

• Notice that we’ve now answered the first three questions with very little pencil work.This just reinforces the notion that if you do the right work up front, you can oftenuse your existing scratch work to pick right answers right off the page.

Page 36: 12-PrepTest 12 Explsdl.keywin.org/5/5/559e448ca144e5a6335f12b48619009d.pdf · KAPLAN LSAT PREP LSAT RELEASED TEST XII EXPLAINED A Guide to the October, 1994 LSAT KAPLAN The answer

LSAT PREP _______________________________________________________________ LSAT Test XII Explained: Section II

34 © K A P L A N

15. (A)Mendel buys exactly two kinds of food? Come on, tell us something we don’t know. Oh,they have: Lara gets exactly two also. But even this sounds pretty familiar; in fact, in theprevious question Lara ended up with two kinds of food. Let’s recap how this could be:Lara doesn’t buy a pretzel (Rule 6) and she can’t buy both a hot dog and a shish kebab(Rule 2). So if she buys two items, one of them must be a fruit cup, choice (A), while theother will be either a hot dog or a shish kebab.

(B) No, we just saw that she could just as easily buy a shish kebab instead.

(C) Under no circumstances, any how, any way, does Mendel buy a fruit cup (Rules 5 and7).

(D) Again, this could be true if Lara buys a shish kebab with her fruit cup (appetizingcombo, eh?), but if she buys a hot dog instead, this choice is history.

(E) Possible, but not if both Lara and Nastassia buy a fruit cup and a hot dog.

• Use your work in previous questions to help answer the current one.

• Don’t worry if a question stem suddenly introduces something that you deducedright from the start, like the “hypothetical” here that Mendel buys exactly two kindsof food. We know from our work up front that that’s not “hypothetical” at all. On thecontrary; you should take this as an affirmation that your deduction is sound.

16. (C)We’ve used Rule 2 often enough to know to immediately cross off “hot dog” for anyonewho gets a shish kebab. So Lara buys a shish kebab but not a hot dog. Since Mendel doesn’tbuy a hot dog (he buys a shish kebab because of Rule 4), and at least one person must buy ahot dog (Rule 3), Nastassia must buy a hot dog. As simple as that.

(A), (E) Lara could buy a fruit cup, but she’s already covered by her shish kebab purchase,so the fruit cup is optional. However, since she can buy a fruit cup and Nastassia alwaysdoes, (E) is no good either.

(B) They’re really stretching things with this choice. Mendel couldn’t buy a fruit cup inQuestion 15, and he can’t buy one here or anywhere else for the same reason (Rules 5 and7).

(D) Actually, Nastassia must buy two kinds of food — a hot dog and her usual fruit cup.

Page 37: 12-PrepTest 12 Explsdl.keywin.org/5/5/559e448ca144e5a6335f12b48619009d.pdf · KAPLAN LSAT PREP LSAT RELEASED TEST XII EXPLAINED A Guide to the October, 1994 LSAT KAPLAN The answer

LSAT PREP _______________________________________________________________ LSAT Test XII Explained: Section II

© K A P L A N 35

17. (C)This is Question 14 reincarnate, only with Rule 2 removed. The first thing we need to do isrethink, and probably eliminate, any deduction that was based on this rule. Uponinspection, we see that the only change is that Mendel can now buy a hot dog. So withoutRule 2, Mendel can now buy three items. However, if he buys a hot dog, Rule 7 still forbidsNastassia from buying one, so she will end up buying only one item, her fruit cup. Sointerestingly, with or without Rule 2, the Mendel and Nastassia combination can buy a totalof exactly four items. Therefore, the key to answering the question rests with Lara. Lara stillcan’t buy a pretzel (Rule 6), she still can buy a fruit cup, but now, without Rule 2, she’s freeto buy both a hot dog and a shish kebab, for a total of three items max. Adding that to thefour items that Mendel and Nastassia buy brings us to a grand total of seven items for $7 asthe maximum amount spent.

• When a question eliminates a rule, always go back to check your original deductionsfor modifications.

• If a rule change necessitates a major reworking of the setup, skip the questiontemporarily: You’re much better off setting up the next game with 5-7 questions upfor grabs than rethinking the current game for a payoff of one measly point. In thiscase, however, the change led to only one minor revision, and the question wasotherwise a repeat of Q. 14, so it’s a question well worth attempting.

Page 38: 12-PrepTest 12 Explsdl.keywin.org/5/5/559e448ca144e5a6335f12b48619009d.pdf · KAPLAN LSAT PREP LSAT RELEASED TEST XII EXPLAINED A Guide to the October, 1994 LSAT KAPLAN The answer

LSAT PREP _______________________________________________________________ LSAT Test XII Explained: Section II

36 © K A P L A N

Game 4 — Chemicals in Flasks(Q. 18-24)

The Action: This game is the latest in a series of games about the steps in a process —games that aren’t particularly amenable to diagramming. Four flasks are filled withdifferent colored chemicals, and the “action” of the game involves combining the contentsof two flasks at a time, a process that results in a chemical of a new color. The rules specifywhat colors result, depending on which flasks are combined. The Key Issues are:

1) When two flasks are combined, what color will the resulting mixture be?2) When you’re given some color information, what combination or set of combinationscould or must have occurred to bring about that particular color scheme?

In other words, some questions are asking you to look forward in the process and follow thesteps through to a result, while others — generally the tougher ones — tell you what theresult of the process was, and ask you to work backwards to reconstruct the process.

The Initial Setup: We’re given the initial colors of the chemicals in each flask, so you maywant to jot that down for reference. There’s little more we can do until the steps in theprocess are described:

1 2 3 4red bl gr or

Before the opening ¶ is over, we become privy to the major action of the game: When thechemicals in two flasks are combined, the two chemicals become one mixture of a differentcolor. No need to write anything — this procedure will be stuck in your mind for the restof the game. (It had better be, because it governs the entire game.) What’s important is thatyou think carefully about the implications of this action.

Visualize the four flasks — mentally make them glass, so that you can “see” the colors, andbe sure to make them big enough so that each can hold the full contents of another flask.Now imagine that you’re picking up any one of the flasks and pouring it into another.(Oops — don’t spill!) What happens? Well:

• One of the flasks is now empty.• Another holds more liquid and, as we can see by previewing the indented rules, the

liquid has probably changed color.• As for the other two flasks — well, hey, they’re the same as they were.

Unchanged — same color, same quantity. Thinking through this process, and making itreal for yourself, is central to what follows.

Page 39: 12-PrepTest 12 Explsdl.keywin.org/5/5/559e448ca144e5a6335f12b48619009d.pdf · KAPLAN LSAT PREP LSAT RELEASED TEST XII EXPLAINED A Guide to the October, 1994 LSAT KAPLAN The answer

LSAT PREP _______________________________________________________________ LSAT Test XII Explained: Section II

© K A P L A N 37

The Rules:

1) Rule 1 clears up a major question that the preceding discussion raises, namely: Whathappens after the two flasks are mixed? The answer: Nothing. The resulting liquid is to beleft alone. (Let’s hope this science student gets a good grade for this brilliant experiment.)

2) Next we get the rules pertaining to the mixing. When the chemicals in flask 1 and flask 2are combined, the resulting mixture is red. You may wish to shorten this to something like“1 + 2 = red” on your page.

3) When the chemicals in flask 2 and flask 3 are combined, the resulting mixture is orange.“2 + 3 = orange.”

4) When the chemicals in flask 1 and flask 3 OR the chemicals in 3 and 4 are combined, theresulting mixture is green. “1 + 3 = green, 3 + 4 = green.”

5) When the chemicals in flask 1 and flask 4 or the chemicals in 2 and 4 are combined, theresulting mixture is blue. “1 + 4 = blue, 2 + 4 = blue.”

Of course, you might choose not to jot the rules down at all, figuring to consult them aswritten on the page. In any event, a typical feature of process games is that one mustcommit, at the outset, to having to go back and check the rules periodically. Don’t fight it;just be sure that you’ve attended to the rules in such a way that you can access theirinformation readily.

Key Deductions: One worthwhile result of Step Four of the Kaplan Approach should bethe realization that all six ways of combining two flasks are accounted for by the rules.

Another, even more important deduction, comes from carrying our mental picture of thiscrazy process a bit further. When last we saw our flasks, one was empty, one had changedcolor (because two flasks were combined in it), and two were unchanged. What else canhappen?

Only two things: Either the project ends right there; or the remaining two flasks arecombined for a corresponding color change and then the project ends, because no othermixings are thereafter possible.

That’s it. Let’s see how this works in the questions.

Page 40: 12-PrepTest 12 Explsdl.keywin.org/5/5/559e448ca144e5a6335f12b48619009d.pdf · KAPLAN LSAT PREP LSAT RELEASED TEST XII EXPLAINED A Guide to the October, 1994 LSAT KAPLAN The answer

LSAT PREP _______________________________________________________________ LSAT Test XII Explained: Section II

38 © K A P L A N

Final Visualization: Not much more than a condensed version of the rules:

1 2 3 4red bl gr or

1+2 = red 2+3 = orange 1+3, 3+4 =blue 1+4, 2+4=green

The Big Picture:

• In a process game, you need to take time during, and after, working through therules, to make certain that you understand the basic process thoroughly. Forexample, mentally working through one combination of chemicals will alert you tothe concept that after one combination, the four flasks with four chemicals willbecome three flasks with three chemicals (and one empty), and a subsequentcombination will give us two flasks with two chemicals.

• If this or any game throws you for a loop, and you can’t seem to “lock in” to the wayit works, skip it. Go for the easier, more accessible points, confident that you’reusing your time in the best possible way (and, most likely, that you’re gaining timeto be spent on this toughie later on). Luckily, this game came last in the section, butremember, on your LSAT, a game like this could come earlier, and you don’t have todo the games in the order presented.

• All Logic Games’ points are created equal, but don’t attempt to treat all games thesame. Some demand more time than others; try to put in that time, but not at theexpense of the really easy games and points elsewhere in the section.

The Questions:

18. (D)We’ve actually already conceptualized this process (see above, beginning around the ¶symbol). What we did not do is work out the possible color schemes in detail. That’s what’scalled for now. Might as well start at the beginning. Consult the rules and use your pencilto jot down your notes:

1 + 2 = red. 3 remains green and 4 remains orange. “Red green orange” not a choice. 1 + 3 =blue. 2 remains blue and 4 remains orange. “Blue blue orange” not a choice. 1 + 4 = green. 2remains blue and 3 remains green. “Green green blue” not a choice.

Phew! Continuing:

2 + 3 = orange. 1 remains red and 4 remains orange. “Orange orange red” not a choice. 2 + 4= green. 1 remains red and 3 remains green. Oh, for the love of . . . Hurray! “Green greenred” is choice (D). For the record, “Blue blue red,” the product of combining 3 + 4 whileleaving 1 and 2 as they were, would also have been an acceptable answer choice.

Page 41: 12-PrepTest 12 Explsdl.keywin.org/5/5/559e448ca144e5a6335f12b48619009d.pdf · KAPLAN LSAT PREP LSAT RELEASED TEST XII EXPLAINED A Guide to the October, 1994 LSAT KAPLAN The answer

LSAT PREP _______________________________________________________________ LSAT Test XII Explained: Section II

© K A P L A N 39

• What can we say? Sometimes you just have to Work It.

• In this case, working out all six possibilities for the three flasks, and leaving themclearly marked in your test booklet, was probably a good idea. You had to work themall out anyhow; might as well leave them nearby just in case.

19. (C)Now we have two experiments to trace. Alas, we’re not told much, but think about it: Therearen’t many possibilities alive in here! If (for example) the student combines 1 and 2, thenthe second experiment will have to be “3 + 4.” Get it? And if he starts by combining twoother flasks. . . well, in the end, there are only three possibilities.

• “1 + 2” and “3 + 4” in either order. They’ll result in red and blue, respectively. (Thatturns out to be the correct answer.)

• “1 + 3” and “2 + 4” in either order, with blue and green resulting.• “1 + 4” and “2 + 3” in either order, with green and orange resulting. So the correct

answer could also have been “blue, green” or “green, orange.” But they chose “blue,red,” choice (C).

• The reason you spend a lot of time, early in the game, thinking through the processin detail, is questions like this.

20. (B)At this point, things get hairy. (I can hear you now, dear reader: “Whaddaya mean at thispoint?!”) We’re given a result and we have to reason backward from it.

Take it one step at a time. One experiment is done — that is, one pair of flasks are mixed —and no red remains. Well, we started out with a red flask, flask 1. Something has to becomeof that red chemical. Well, flask 1 must be part of the experiment: It has to be mixed withanother flask, and that will remove the red. What’s more, it can’t be mixed in a way that willresult in red. (Remember, no red remains!) That means that the experiment cannot be “1 + 2”(keep checking your list), because 1 + 2 leads to red. And so the experiment must be either:

• “1+3,” which becomes blue; 2 stays blue and 4 stays orange. Is “blue, blue, orange” ananswer choice? Drat! That leaves:

• “1+4,” which becomes green; 2 stays blue and 3 stays green. Where’s “blue, green,green”? It has to be there, and it is: (B).

• Avoid panicking! Think things through one step at a time. And if nothing’shappening. . . move on to better things, another question or another game.

Page 42: 12-PrepTest 12 Explsdl.keywin.org/5/5/559e448ca144e5a6335f12b48619009d.pdf · KAPLAN LSAT PREP LSAT RELEASED TEST XII EXPLAINED A Guide to the October, 1994 LSAT KAPLAN The answer

LSAT PREP _______________________________________________________________ LSAT Test XII Explained: Section II

40 © K A P L A N

21. (A)Sort of a switch on Q. 20; here, exactly one flask contains blue after the single experiment.(And notice that the question is “What must be the colors in the two flasks not involved inthe experiment?” That “must” tells you that the whole thing can be worked out withcertainty.)

Think about the experiments as laid out in the rules and the questions demand for blue.Two of the mixes result in blue: “1 + 3” and “3 + 4.” But either one of those would violatethe question stem, because flask #2 is blue to begin with, and that would result in two flaskswith blue. That means that the single blue flask after the experiment has to be the one thatoriginally contained blue — flask #2.

Let’s continue searching — and here’s where working out all of the possibilities during Q.18, and keeping them tidy for later referral, pays off. Doing so allows you to see quickly that,having eliminated “1 + 3” and “3 + 4” from consideration, as well as every pairing thatinvolves flask 2 (which, as we’ve just said, has to be left alone — we need that single blueflask), the only remaining pairing is “1 + 4.” That mix results in green; flask 3 remainsgreen; and “green, green” is (A).

• Read stems carefully! We often proceed differently when asked “what can be true?”as opposed to “what must be true?”

22. (E)Again, one step at a time: and remember, this ends up as a “could be true.” Afterexperiment 1, we have exactly one orange. Well, if flask 4, which begins orange, is involvedin the experiment, then we’ll be left with zero orange flasks. (4 + nothing = orange; checkyour list.) And if 4 is left alone but experiment 1 is “2 + 3,” then we’d be left with twoorange flasks (2 + 3 = orange). Neither of those is acceptable.

Therefore, what is not only acceptable but inevitable is that flask 4 won’t participate inexperiment 1, thus standing as our single orange flask. We’ll need flask 4 for experiment 2.Meanwhile, the first experiment will be a pair that leads to a color other than orange. Thecandidates? Check your list or work it out: “1 + 2,” which will become red, and “1 + 3,”which will become blue.

The important thing is to move on to experiment 2 which, as we’ve already seen, will mixflask 4 with one of the two remaining flasks, #2 or #3 (whichever one wasn’t mixed withflask 1 in the first experiment). So either “2 + 4” or “3 + 4” will be the right answer, and thetestwriters chose the latter as choice (E).

• Keep an eye on your answer choices, and try whenever possible to toss out wronganswers. For instance, the moment you see that flask 4 cannot participate inexperiment 1 and is needed for experiment 2, you can toss out whatever choices failto mention flask 4. That’d leave you with (C) and (E), a one-in-two guessing shot.

Page 43: 12-PrepTest 12 Explsdl.keywin.org/5/5/559e448ca144e5a6335f12b48619009d.pdf · KAPLAN LSAT PREP LSAT RELEASED TEST XII EXPLAINED A Guide to the October, 1994 LSAT KAPLAN The answer

LSAT PREP _______________________________________________________________ LSAT Test XII Explained: Section II

© K A P L A N 41

23. (E)This one’s pretty easy, certainly easy compared to the ones preceding it. If after oneexperiment none of the flasks contains orange, then clearly flask 4, our original orangeflask, must have been mixed with some other flask, and that’s all that (E) says.

• Never despair. Easy questions can and do come towards the end of a game.

24. (D)After that brief respite, another pain in the neck. (Well, if you’ve come this far, maybe it’snot so bad.) We’re told that at the end of experiment 2, one of the two filled flasks containsorange. Well, one and only one mixture leads to orange — that’s “2 + 3.” So one experimentmust have been “2 + 3,” meaning that the other experiment must have been “1 + 4,” whichresults in green. So green is in the other non-empty flask, choice (D).

• A final thought: Never be surprised to stumble upon a game that’s really tough.After all, one of the four games in your scored Logic Games section is going to berated difficult, meaning that most examinees aren’t expected to do well on it. Now,that high-difficulty game may not be the one that you have trouble with, but be thatas it may: Virtually no one gets through an entire games section unscathed. There aregoing to be valleys. . . but there will be peaks as well. If you keep in mind that a greatLSAT score doesn’t require that you get every LG question right (by a long shot),and if you remember that the lion’s share of points will be adequate, then you’ll bein the best possible frame of mind to attack every game, and the section as a whole,and come out a winner.

Page 44: 12-PrepTest 12 Explsdl.keywin.org/5/5/559e448ca144e5a6335f12b48619009d.pdf · KAPLAN LSAT PREP LSAT RELEASED TEST XII EXPLAINED A Guide to the October, 1994 LSAT KAPLAN The answer

42 © K A P L A N

SECTION III:

READING COMPREHENSION

Page 45: 12-PrepTest 12 Explsdl.keywin.org/5/5/559e448ca144e5a6335f12b48619009d.pdf · KAPLAN LSAT PREP LSAT RELEASED TEST XII EXPLAINED A Guide to the October, 1994 LSAT KAPLAN The answer

LSAT PREP ______________________________________________________________ LSAT Test XII Explained: Section III

© K A P L A N 43

Passage 1 — Modern Architecture(Q. 1-7)

Topic and Scope: Modern architecture; specifically, how Wagner’s book illustrates thedisparity between the practical emphasis of today’s architects and the original ideals ofmodern architecture’s founders.

Purpose and Main Idea: Author wants to show that architecture as it’s practiced today(“emphasizing practical and technical issues”) has veered sharply away from the originalaesthetic ideals expressed in Wagner’s classic text, Modern Architecture.

Paragraph Structure: ¶ 1 identifies the problem: While opponents criticize the “cost-efficiency and utility” of today’s architects, the fact is that “the failures of modernarchitecture” aren’t the fault of its founders’ ideals. ¶’s 2-4 examine the views and ideals ofOtto Wagner, a representative “founder” of modern architecture.

¶ 2 focuses on Wagner’s call for a new style based on modern technologies, ending with amomentary suggestion that Wagner spearheaded a totally practical “dogma.” But it’s onlymomentary, because “the picture was more complex.” ¶ 3 explains that, beyond hisembrace of modern technology, Wagner stressed a symbiotic relationship betweenengineering and architecture, in which art (aesthetics, in other words) “was to exercise thecontrolling influence.” ¶ 4 focuses on Wagner’s roots in the Classical tradition. Note theexample provided in the last sentence: While Wagner was rationally committed to moderntechnology, he was emotionally grounded in “the great works of the Italian Renaissanceand Austrian Baroque.”

The Big Picture:

• Orient to the broad strokes in every author’s argument — they’re always few innumber: Here, we first get the contrast between today’s architecture and Wagner’sideals; and second, Wagner’s dual commitment to technology and art, with anemphasis on the latter. Those ideas will score points.

• In “book review” passages, remember to differentiate between the viewpoints of thepassage author and those of the book author (here it’s Wagner).

The Questions:

1. (D)(D) mentions Wagner’s book and paraphrases his concern that modern architectureincorporate modern technology while remaining faithful to aesthetic standards. Only (D)encompasses the thrust of every paragraph.

(A) repeats a point made in the passage’s opening statement, which is then left behind. Thefollowing ¶’s focus on Wagner’s ideas, not on criticism of modern architecture.

(B) contradicts the passage — ¶ 1 implies that most modern architects have downplayedthe aesthetic in favor of the practical.

Page 46: 12-PrepTest 12 Explsdl.keywin.org/5/5/559e448ca144e5a6335f12b48619009d.pdf · KAPLAN LSAT PREP LSAT RELEASED TEST XII EXPLAINED A Guide to the October, 1994 LSAT KAPLAN The answer

LSAT PREP ______________________________________________________________ LSAT Test XII Explained: Section III

44 © K A P L A N

(C) is outside the scope of the passage, whose author never describes Modern Architecture asanything like “a chronicle” (i.e. an historical narrative). Wagner’s book sounds more like acall to arms.

(E) focuses narrowly on a point made at the end of ¶ 2.

• A sharp, take-charge attitude with the choices always pays off. For example, note thatonly three choices mention Wagner’s book. Give (A) and (B) the briefest glance, thenfocus your energies on the others.

2. (E)The stem steers you toward the answer: Engineers are discussed in ¶ 3, so go there andread for the gist, which is that the architect must “develop the skills of the engineer withoutlosing the powers of aesthetic judgment....” (E) paraphrases that.

(A) misses the point, the very spirit of the thing: The roles of architect, engineer, and artistneed to be combined.

(B) Au contraire: The engineer’s capabilities are described as purely technical.

(C) On the contrary, ¶ 3 describes the architect, who assumes the roles of engineer andartist, as predominant.

(D) Au contraire once again.

• Don’t let wordy choices suck you in. Roughly prephrase the answer. Go from thestem to the passage, pick up the gist, and then scan the choices with the author’sideas in mind, dismissing those that don’t begin to click. The only one that’s worthyour extra time and thought is the correct one!

3. (B)This idea counters the gist of ¶ 4: Wagner revered the great works of the Renaissance andBaroque, and saw no inconsistency between historical forms and modern design — thecircular church design he favored was based on a Renaissance type. That lack ofinconsistency is pretty much the whole point, isn’t it? (See (D) in Q. 1.)

(A) repeats an idea attributed to Wagner earlier in the same paragraph.

(C) combines the gists of ¶’s 3 and 4.

(D) repeats info in the last sentence, which also echoes the gist of ¶ 2: Wagner embracedmodern technology, knowing that preindustrial technology wasn’t appropriate for today’sbuildings.

(E) also combines the last sentence and ¶ 2: Wagner knew there was no way back toprevious “social” conditions, and he saw the “political and technological revolutions ofthe nineteenth century” as creating imperatives that the modern architect had to follow.

Page 47: 12-PrepTest 12 Explsdl.keywin.org/5/5/559e448ca144e5a6335f12b48619009d.pdf · KAPLAN LSAT PREP LSAT RELEASED TEST XII EXPLAINED A Guide to the October, 1994 LSAT KAPLAN The answer

LSAT PREP ______________________________________________________________ LSAT Test XII Explained: Section III

© K A P L A N 45

• Unusually wordy choices, and an EXCEPT format to further complicate things: allsuggestions that you might want to move on to potentially easier questions. But holdon — as it turns out, (B) stands out.

• Use the implied logic of the stem. If one choice is LEAST likely to be a Wagnersentiment, chances are he would agree with the other four — an alternative way ofproceeding towards (B).

4. (C)The trick here is to be aware of the larger context; it’s the first sentence of ¶ 3 that suppliesthe answer. Although Wagner embraced modern technology (as the quotes in ¶ 2 makeclear), “the picture [¶ 3 explains] was more complex....” (C) catches on.

(A) The reverse would be true: The quotes show Wagner’s advocacy of moderntechnology, which becomes the dominant concern of modern high-rise building designers.

(B) Also inconsistent with the passage. Proponents of Wagner’s ideas would be sure to citehis emphasis on aesthetic ideals.

(D) is outside the scope — absolutely never suggested.

(E) is similar to (B), and wrong for the same reason. The content of ¶ 2 gives a veryincomplete view of Wagner’s ideals. This is the one you’d pick if you forgot which ¶ wascalled for. Which reminds us:

• You gain nothing if, in the interests of “saving time,” you zip through a questionstem and miss or mess up a key clue (as, for instance, the reference to ¶ 2 here). Thisadvice holds for the other sections too, incidentally.

5. (B)A direct paraphrase of the passage’s first two sentences — which are going to be keysentences in virtually every passage!

(A) distorts the passage by mixing aesthetic demands in with cost-efficiency and utility.The former, according to ¶ 1, are not a primary concern of modern architecture.

(C) The passage never distinguishes cost-efficiency from other practical concerns.

(D) distorts material in ¶ 4. The circular church was a design justified by Wagner, not arepresentative example of modern architecture which, we can infer, does not look tohistorical models.

(E) Au contraire: The opening lines imply that aesthetic value is missing in many modernbuildings.

Page 48: 12-PrepTest 12 Explsdl.keywin.org/5/5/559e448ca144e5a6335f12b48619009d.pdf · KAPLAN LSAT PREP LSAT RELEASED TEST XII EXPLAINED A Guide to the October, 1994 LSAT KAPLAN The answer

LSAT PREP ______________________________________________________________ LSAT Test XII Explained: Section III

46 © K A P L A N

• Hot lines in any passage are the first third, plus the first and last sentences of eachparagraph. Those lines always yield points — here, for instance, they unlock Q.’s 1, 3,4, 5, and 7!

6. (E)Note the word-for-word parallel between ¶ 4 and choice (E): The “tension” betweenClassicism and modern technology (E) is broached in the sentence preceding the citedexample of the church design.

(A) distorts the passage: Wagner is never described as introducing technologicalinnovations. In any case the emphasis in the church plan is on Wagner’s love of classicalmodels.

(B) On the contrary, Wagner loved those historical forms.

(C) A bit tricky, perhaps. Wagner’s “true inspiration” was the older designs, but thechurch plan was a design, not an actual modern building.

(D) Also tricky, but distorts the passage, which never implies that Wagner was a mere“imitator” of the Renaissance and Baroque.

• It’s imperative here to go back to the passage and let the passage structure reveal theanswer. The church is cited as an example of something — something stated in thepreceding lines, right? If you don’t follow the structure, you can end up choosingfuzzily tempting choices like (C) and (D).

7. (E)The “assertion” to which (E) refers comes in ¶ 1: “However,” (a great keyword!) OttoWagner’s text shows that “the failures of modern architecture cannot be blamed on the ideals of itsfounders.” ¶’s 2-4 then support the assertion by discussing Wagner’s seminal work.

(A) is imprecise. No particular debate is dealt with and no history is summarized.

(B) comes closer, but the author never refers to Wagner’s views as a “traditional argument.”

(C) The only “recent approach” referred is the practical emphasis of today’s architects. Butthat’s not the focus in ¶’s 2-4, for sure.

(D) distorts the passage. The author isn’t an opponent of modern architecture. Heacknowledges opponents’ criticism, agreeing that today’s architecture is flawed, but herescues modernism from its critics by pointing to Wagner.

• Don’t debate over imprecise, dirty choices: (B), (C), and (D), for instance, will surelytie you up if you let them. Only one choice will be clean and air-tight. Trash theothers.

Page 49: 12-PrepTest 12 Explsdl.keywin.org/5/5/559e448ca144e5a6335f12b48619009d.pdf · KAPLAN LSAT PREP LSAT RELEASED TEST XII EXPLAINED A Guide to the October, 1994 LSAT KAPLAN The answer

LSAT PREP ______________________________________________________________ LSAT Test XII Explained: Section III

© K A P L A N 47

Passage 2 — Socioeconomic Achievement of Asian Immigrants(Q. 8-13)

Topic and Scope: The socioeconomic success of Chinese and Japanese immigrants andtheir descendants; specifically, theories about how these groups coped with dauntingconditions in the U.S.

Purpose and Main Idea: The author seeks to combine two distinct and commonly usedsociological theories. The main idea — the idea that links the entire passage together —comes at the beginning of ¶ 2: The mobility of Chinese and Japanese immigrants is bestunderstood by considering both supply-side (cultural) and demand-side (structural)analyses together.

Paragraph Structure: ¶ 1 defines the two theories. Culturally based (a/k/a “supply side”)explanations emphasize the cultural resources that immigrants bring with them to the U.S.Structurally based (“demand side”) explanations stress the “market” conditions in the U.S.that the immigrants must respond to.

¶ 2 starts with the main idea — both analyses are necessary — and then discusses thesupply side analysis, basically comparing the Chinese and Japanese experiences and takingup such issues as how each group immigrated and what happened subsequently. The gistof it is that the Japanese moved ahead a bit faster than the Chinese, but the situation moreor less evened out in time.

¶ 3, predictably, focuses on the structural (demand side) explanation, which stresseseconomic conditions in the host country. The paragraph basically compares the demandfor immigrant labor in “early capitalist” and “advanced capitalist” economies.

The Big Picture:

• Note that the author’s purpose, while not explicit until ¶ 2, was suggested in the firstsentence: She notes that the two sociological views are typically not applied together,implying that she herself would be doing just that.

• The problem with this passage is with the relentless jargon — terms like economicmetaphor, supply side, demand side, human-capital perspective, status attainment,household production, etc etc etc. Whew! Sweat! But wait — Kaplan’s trusty axiomapplies: Key ideas are few in number, and they’re easier than the language suggests.Stay level-headed and in charge, and keep a sense of humor about the usual LSATantics. Anything that’s genuinely important will be explained (often several times)before you’re through.

• Keep things simple. Supply side (cultural) factors refer to things like family andpersonal resources, things that made adjustment to life in the U.S. easier. Demandside (structural) factors refer to existing U.S. economic conditions that immigrantshad to deal with. You don’t need to understand the concepts more deeply than that.

Page 50: 12-PrepTest 12 Explsdl.keywin.org/5/5/559e448ca144e5a6335f12b48619009d.pdf · KAPLAN LSAT PREP LSAT RELEASED TEST XII EXPLAINED A Guide to the October, 1994 LSAT KAPLAN The answer

LSAT PREP ______________________________________________________________ LSAT Test XII Explained: Section III

48 © K A P L A N

The Questions:

8. (E) Your jaw may have dropped at the length of these choices, but we hope you breatheddeeply and plunged in. Pre-phrasing helps: “The proper analysis combines supply anddemand theories.” It also helps if you push aside any choices that don’t start to work in thefirst line or so. (E), unlike all the others, pulls us in, from its first five words: “Only ananalysis that combines...” (E) alone echoes the opening sentence of ¶ 2 and encompasses allparagraphs.

(A) is too narrow; it refers to the second theory, the structurally-based explanation.

(B) has the same problem, in reverse: It focuses on the culturally-based explanation.

(C) focuses narrowly on the nuclear family, a supporting detail in ¶ 2.

(D), like (A), focuses on the structural explanation.

• The lengths of these choices is truly absurd (more LSAT antics)! But note: The firstline or two of (E) gives it away. The other choices are traps. There’s always a short cutwhen you’re confronted with an unruly batch of choices: You just have to look forit....

9. (C)If you correctly associate “supply side” with the immigrant point of view — with, as ¶ 1says, the “qualities immigrant groups bring with them [to the host] market” — then choice(C) must stand out. All the others refer to demand-side elements: economic realities in thehost country.

• Questions like this are legion in passages that focus on a broad contrast. Be ready forthem.

10. (D)The cited sentence — actually the first 4 sentences of ¶ 3 — are talking about changes indemand-side factors: the evolution in the host country from an early capitalist economy toan advanced capitalist economy, which eventually creates the demand for both skilled andunskilled labor (“two immigrant streams”).

(A) “Human capital” is a jargon term that’s irrelevant to ¶ 3. It came up in ¶ 1, in relationto the cultural or supply side perspective.

(B) is confusing; it may be tempting but is inaccurate. The cited sentence mentions theprimary labor market, but seemingly in relation to skilled labor only. The stem asks about“two immigrant streams” — about both skilled and unskilled labor.

(C) focuses wrongly on early immigration. The “two immigrant streams” is a later responseto an advanced capitalist economy.

Page 51: 12-PrepTest 12 Explsdl.keywin.org/5/5/559e448ca144e5a6335f12b48619009d.pdf · KAPLAN LSAT PREP LSAT RELEASED TEST XII EXPLAINED A Guide to the October, 1994 LSAT KAPLAN The answer

LSAT PREP ______________________________________________________________ LSAT Test XII Explained: Section III

© K A P L A N 49

(E) is as imprecise as (B). The passage says nothing about the two immigrant streamshaving any influence on the labor market.

• A good example of a question for which the surrounding context is crucial. Note that(B) and (E), which stay within the cited sentence, miss the point, while (D) looksback and captures the real idea by focusing on the context, the gist of the precedingsentences.

11. (E)The answer echoes the gist of ¶ 1. The author describes the two sociological treatments astypically applied separately. She then asserts at the beginning of ¶ 2 that both should beused together.

(A) is illogical and never implied. No doubt everybody — the author and all sociologists— would be interested in the interaction of causal factors.

(B) Au contraire: The structuralists focus on conditions in the host country, very muchincluding “early capitalist development.” Moreover, nothing implies that moresociologists ascribe to the cultural perspective than to the structural.

(C) True, the author uses the economic metaphors of supply-side and demand-side todescribe the two theories, but there’s no sense that the majority of sociologists woulddisavow those metaphors.

(D) Another distortion. Issues of racial discrimination against immigrants (says ¶ 1) areintegral to both sociological theories, which the author embraces.

• Many had trouble with this question. Hope you didn’t get bogged down too muchby it: The next two are simpler.

12. (A)Inferable from ¶ 2. Chinese immigrants can only be said to have “remained sojourners”(line 32) if they had been such; likewise, if the Japanese “made the transition fromsojourner” (lines 36-37), they must have been sojourners at some point.

(B) Apparently true of Chinese but not of Japanese immigrants.

(C) and (D) are both true of the Japanese, who quickly established small businesses, but lessso of Chinese, who took longer to establish roots.

(E) Technical skills are never mentioned, but are inferably more true of Japanese, whoinitially advanced faster than the Chinese.

• The stem clearly refers to info in ¶ 2, which compares the mobility of Chinese andJapanese immigrants. The gist is that the two groups assimilated at different rates,which eliminates all the wrong choices, leaving (A).

Page 52: 12-PrepTest 12 Explsdl.keywin.org/5/5/559e448ca144e5a6335f12b48619009d.pdf · KAPLAN LSAT PREP LSAT RELEASED TEST XII EXPLAINED A Guide to the October, 1994 LSAT KAPLAN The answer

LSAT PREP ______________________________________________________________ LSAT Test XII Explained: Section III

50 © K A P L A N

13. (A)(A) says it all. The author’s purpose is to combine the two sociological perspectives. Onlyconsidering them together, she says at the start of ¶ 2, “will suffice.”

(B) The verb “challenging” doesn’t work. The author doesn’t challenge either of the twotheories; she wants to use them. Nor is there any reference to a “tentative answer to aquestion.”

(C) distorts the author’s approach. She doesn’t evaluate the theories; she makes use of them.

(D) also distorts. No “differences” are resolved. The two approaches involve distinctperspectives but they’re not opposed to each other.

(E) is wide of the mark. The only mention of “group achievement” comes in ¶ 2, and thefocus there is on two immigrant groups, not one. The bigger concern, in any case, is toapply the sociological theories.

• Purpose questions whose choices are written in abstract language can be confusingeven if the choices are as brief as they are here. Pre-phrasing is the best way to avoiddistraction by superficially plausible, but seriously flawed, answers.

Page 53: 12-PrepTest 12 Explsdl.keywin.org/5/5/559e448ca144e5a6335f12b48619009d.pdf · KAPLAN LSAT PREP LSAT RELEASED TEST XII EXPLAINED A Guide to the October, 1994 LSAT KAPLAN The answer

LSAT PREP ______________________________________________________________ LSAT Test XII Explained: Section III

© K A P L A N 51

PASSAGE 3 — Legal Reasons(Q.14-20)

Topic and Scope: American and English legal systems; specifically, America’s reliance on“substantive” reasons and England’s reliance on “formal” reasons.

Purpose and Main Idea: Author’s purpose is to contrast the different legal reasonsemployed by the American and English legal systems; this is a “storytelling” passage, inwhich the author simply lays out a fundamental difference between the American andEnglish legal systems, so there is no specific main point or idea. (No surprise, then, thatthere’s no specific main point or idea question.)

Paragraph Structure: ¶ 1 hinges on the Keyword “Although” (line 1), which signals thatthe author’s about to describe a basic difference between the otherwise similar Americanand British legal systems — America’s reliance on substantive legal reasons and England’sreliance on formal legal reasons. The ¶ ends without clear-cut definitions of those terms,but you knew (didn’t you?) that they would be forthcoming.

¶s 2 and 3, as anticipated, describe substantive and formal legal reasons. Substantivereasons — as the example of the World War II jeep in the park suggests — are based on thespirit rather than the letter of the law. Formal reasons — as the example of the will implies— are based on the letter of the law. You don’t really have to understand things much moredeeply than that.

¶ 4 wraps things up with an assertion of problems stemming from “extreme examples” ofeach tradition: The English system’s emphasis on strict adherence to past legal traditionshas inhibited its development, while the American system’s emphasis on judicialinnovation has led to situations in which established laws are ignored.

The Big Picture:

• You might want to leave a “storytelling” passage like this one for later in the section.Why? Although topic and scope are revealed early in ¶ 1, the author’s purpose is notentirely clear until you finish the passage. You might have predicted that the authorwas going to limit himself to a simple contrast without really taking a position for oragainst anything, but you don’t actually know that for sure till the end.

• Be on the lookout for passages that contrast two (or more) things: points of view,cases, theories, systems, etc. Such passages always contain questions that hinge on afirm understanding of each of the things being compared.

• A “firm” understanding doesn’t mean detailed or in depth: It means clear and broad.Here, as long as you get that substantive = U.S. = “spirit of law” = more or lessliberal, while formal = England = “letter of law” = much more conservative, you doO.K. Don’t try to read as if you were a legal scholar, read as who you are: a laypersonwho has less than 10 minutes to get the gist of the thing and answer a set ofquestions on it!

Page 54: 12-PrepTest 12 Explsdl.keywin.org/5/5/559e448ca144e5a6335f12b48619009d.pdf · KAPLAN LSAT PREP LSAT RELEASED TEST XII EXPLAINED A Guide to the October, 1994 LSAT KAPLAN The answer

LSAT PREP ______________________________________________________________ LSAT Test XII Explained: Section III

52 © K A P L A N

The Questions:

14. (C)Straightforwardly encompasses the correct topic, scope, and purpose of the passage.Should be a slam dunk.

(A) is beyond the scope, way too broad. The text focuses on one specific difference betweenthe legal systems, not multiple ones. Moreover, the text doesn’t go into any specificsimilarities between the two, it just suggests that they exist.

(B) is less broad than (A), but not by much. (B)’s noun is off. A “reevaluation” wouldsuggest that the author’s got a definite point of view; this author’s purpose, however, isdescriptive rather than argumentative. Secondly, what’s the initial evaluation of which thisis a “re-”? Finally, this choice doesn’t refer at all to the topic and scope of the text.

(D) also hinges on a poor noun. The text describes; it doesn’t explain. And, again, there’s noreference to the American and English legal systems, let alone to a basic difference betweenthem.

(E), like (B) and (D), fails to mention the American and English legal systems. It alsomisrepresents the author’s purpose, which is to sharply contrast two types of legal reasons.

• Wrong answers to global questions are often wrong for subtle reasons that mayescape you in a too-quick reading. Read all of the choices carefully, and alwaysconsider topic, scope, and purpose before you endorse any choice.

15. (E)English legal decisions are based on formal reasons — i.e., the letter of the law. Therefore,English judges would find the veterans group guilty on the grounds that it violated the“stipulated requirements” of the statute, which forbids placing vehicles in public parks.

(A) goes beyond the scope. The text never says or implies that the English — or, for thatmatter, the American — legal system is concerned with making examples out oflawbreakers in order to dissuade others from engaging in similar acts.

(B), (C), and (D)’s focus on substantive reasons would interest American judges. But as lines29-32 point out, most U.S. courts would be unlikely to find the veterans group guilty ofviolating the statute because it had complied with the substantive purpose of the statute —to keep the public park quiet and safe.

• Whenever possible, “pre-phrase” answers to questions. Doing so will focus yoursearch among the answer choices. In this case, pre-phrasing should have led youdirectly to (E), the only choice that focuses on a concern of English judges.

Page 55: 12-PrepTest 12 Explsdl.keywin.org/5/5/559e448ca144e5a6335f12b48619009d.pdf · KAPLAN LSAT PREP LSAT RELEASED TEST XII EXPLAINED A Guide to the October, 1994 LSAT KAPLAN The answer

LSAT PREP ______________________________________________________________ LSAT Test XII Explained: Section III

© K A P L A N 53

16. (E)¶ 3 suggests that substantive reasons are only taken into account when judges decide thatexceptions to formal reasons, or “stipulated requirements,” are permissible because thelegality of a law is still open to question. In the case of a will, then, substantive reasonsabout its validity would be taken into account only if a judge reasoned that the formalprovision (that a will must be witnessed in writing) could be waived because its legalityhad not yet been established. (E) alone outlines such a scenario.

No other choice waives the “stipulated requirement” that a will must be witnessed inwriting.

• If a question baffles you, scan the answers for a choice that looks different from theothers, as (E) looks here. Then evaluate that choice on its own merits.

• Many examinees agreed that this question was the toughest in the set. Those who gotbogged down and/or discouraged while struggling with it did themselves no favor.Remember that there’s no rhyme or reason to the layout of RC questions: The easierones could come anywhere, so go find and attack those, and come back to a“toughie” a bit later, your confidence (and points) intact.

• Note, too, that a later question — #19 — is going to enter the same area as this one,and may shed some light that will render #16 easier in retrospect. Ya never know.

17. (D)The passage contains no “chronology of historical developments.” Indeed, the onlyreferences to historical matters — and they’re rather vague — surface in ¶’s 1 and 4, whenthe author talks about the “traditional” views of the American and English legal systemsand the legal consequences of using substantive and formal reasons, respectively. Onereference to World War II does not a historical chronology make.

(A) The entire text is built around a contrast of the different legal reasons — substantive vs.formal — used by the two legal systems.

(B) is also used throughout the passage. ¶ 1, for example, contains a number ofgeneralizations. ¶’s 2, 3, and 4 begin with general statements that are then supported withdetails.

(C) ¶’s 2 and 3 explain substantive and formal legal reasons, respectively.

(E) refers to the hypothetical jeep in ¶ 2 and the hypothetical will in ¶ 3.

• In all/EXCEPT questions, look for the choice that’s not true.

Page 56: 12-PrepTest 12 Explsdl.keywin.org/5/5/559e448ca144e5a6335f12b48619009d.pdf · KAPLAN LSAT PREP LSAT RELEASED TEST XII EXPLAINED A Guide to the October, 1994 LSAT KAPLAN The answer

LSAT PREP ______________________________________________________________ LSAT Test XII Explained: Section III

54 © K A P L A N

18. (A)Nicely summarizes — albeit in abstract terms — the function of the last ¶. Lines 51-59describe the legal consequences for America and England, respectively, of “extremeinterpretations” of substantive and formal reasons.

The four remaining choices all misrepresent the last ¶’s function, which isn’t to commenton the validity of legal scholars’ methods or views. It certainly isn’t to comment negativelyon legal scholars, and notice that inaccuracy or blunder is a common theme among all fourwrong answers. In ¶ 3, the author simply mentions legal scholars in order to introduce thepoint that extreme interpretations of substantive and formal reasons have had a significantimpact on the American and English legal systems, respectively.

• Questions ask about the why of a detail or paragraph; that’s why we’re so interestedin “reading for paragraph sense” in our initial attack on the passage. Keep theauthor’s purpose in mind during the questions.

19. (C)Deftly paraphrases lines 43-48, which indicate that a substantive interpretation of a law ispermissible under the English legal system if the legality of that law has yet to beestablished.

(A) and (B) distort the passage. Prevailing social conditions (A) and legislative preferences(B) are concerns taken into account in the American legal system, but don’t warrant ajudgment on substantive grounds under the English system.

(D)’s concern with the impact of an English offender’s arguments is beyond the scope of thediscussion of the English system.

(E) is an au contraire choice. Under the English system, once a legal rule “has been clearlyestablished” by past legal decisions, a substantive interpretation of a law is no longerpermissible.

• Your work on one question can often help you to choose the correct answer toanother, related question. Try to work on “paired” questions — like #’s 16 and 19 inthis set — one after the other. At the very least, try to recognize such pairs by keepingin mind that multiple questions often cover the same ground.

Page 57: 12-PrepTest 12 Explsdl.keywin.org/5/5/559e448ca144e5a6335f12b48619009d.pdf · KAPLAN LSAT PREP LSAT RELEASED TEST XII EXPLAINED A Guide to the October, 1994 LSAT KAPLAN The answer

LSAT PREP ______________________________________________________________ LSAT Test XII Explained: Section III

© K A P L A N 55

20. (A)Paraphrases lines 13-16, and picks up on the idea of substantive reasons’ going beyond theletter to the spirit of the law.

(B) Au contraire, under the American system, substantive reasons don’t have to be writteninto a law in order to be “relevant” to its “application,” as the hypothetical example of thejeep in the park (lines 19-32) demonstrates. Even under the English system, substantiverules may sometimes not be relevant to a law’s application, even if written into the lawitself (lines 33-36).

(C) is another au contraire choice. Lines 13-15 say that substantive reasons are often not legalin nature; and, under the English system, they’re often not relevant to the application oflaws at all.

(D) distorts a point brought up in lines 55-59. According to these lines, some judges in theAmerican system only have used substantive reasoning to ignore the law.

(E) also distorts the passage. Substantive reasons are central to the American legal system,whereas formal reasons are central to the English system — that’s in lines 4-6.

• The answer to explicit text questions is always in the passage itself, but recognizethat these answers, too (like choice (A) here) have to fit in line with the spirit of thepassage. That doesn’t mean that you shouldn’t refer back to the text — you should;but be guided by the overall scope and purpose of the passage at the same time.

Page 58: 12-PrepTest 12 Explsdl.keywin.org/5/5/559e448ca144e5a6335f12b48619009d.pdf · KAPLAN LSAT PREP LSAT RELEASED TEST XII EXPLAINED A Guide to the October, 1994 LSAT KAPLAN The answer

LSAT PREP ______________________________________________________________ LSAT Test XII Explained: Section III

56 © K A P L A N

PASSAGE 4 — Serotonin(Q.21-27)

Topic and Scope: The brain and what it knows about carbohydrate consumption;specifically, the link between serotonin levels in the brain and the body’s (the brain’s)desire for carbohydrates. Serotonin is definitely the “star” of this particular passage.

Purpose and Main Idea: Author’s purpose is to describe the link between serotonin andcarbohydrate desire; the author’s specific main idea is that, in general, the more serotoninthat is present in the brain, the less likely the body is to crave carbohydrates, and vice-versa. It’s an inverse relationship, one that you need to be clear on. (But that’s about all youneed to be clear on!)

Paragraph Structure: ¶ 1 sets out the basic issue at hand — the link between serotoninlevels and the body’s desire for carbohydrates, suggesting that there is an inverserelationship between the two. ¶ 2, once you cut through all of the scientific factology, saysthat the production of serotonin itself is linked to the amount of carbohydrates that thebody is receiving. (As we’ll see, the role of tryptophan in the passage — as opposed to itsrole in the bloodstream — is mainly to generate wrong answer choices.)

¶ 3 reverts to the original issue of how serotonin levels affect the body’s craving forcarbohydrates. Rats, we learn, are affected in much the same way as humans: increasedserotonin levels make them desire carbohydrates less. ¶ 4 brings us full circle to ¶ 1: Drugsthat enhance serotonin levels decrease the human body’s desire for carbohydrates, whilethose that depress serotonin levels increase it. Yadda yadda yadda; same-o same-o. If youever doubted or were unaware of the redundancy that can often drive a Reading Comp.passage, especially a science passage, we hope that this one was an eye-opener.

The Big Picture:

• Like most science passages, this one contains a lot of details, but the passage’s mainidea — the inverse relationship between serotonin levels and the body’s desire forcarbohydrates — is evident from the outset, is straightforward, and never reallydevelops further. Chances are you saved this passage till last; hope you got apleasant surprise from how manageable it was.

• Underneath all of the scientific jargon is a straightforward, easy-to-understand idea,so don’t let yourself be thrown by the complex prose. And be sure that youunderstand the main idea, because many of the questions in science passages willreward you for doing nothing more than grasping it.

The Questions:

21. (D)The only choice that captures the passage’s topic, scope, and purpose. Like #14 earlier, aslam dunk.

Page 59: 12-PrepTest 12 Explsdl.keywin.org/5/5/559e448ca144e5a6335f12b48619009d.pdf · KAPLAN LSAT PREP LSAT RELEASED TEST XII EXPLAINED A Guide to the October, 1994 LSAT KAPLAN The answer

LSAT PREP ______________________________________________________________ LSAT Test XII Explained: Section III

© K A P L A N 57

(A) distorts the passage. The text is about the link between serotonin levels and the body’sdesire for carbohydrates. What the body desires is not necessarily what it needs.(Incidentally, as far as we know from the passage, serotonin exists in the central nervoussystem; it’s tryptophan that lives in the blood.)

(B) focuses on a detail in ¶ 4. Can’t be what #21 is looking for.

(C) focuses on a detail in ¶ 3. Ditto.

(E) focuses on a detail in ¶ 2. Ditto. How can tryptophan masquerade as the star of thisshow?

• The author’s main idea must be consistent with the text’s topic, scope, and purpose,and the answer cannot, simply cannot, grab onto a detail or single paragraph like adog to a bone.

22. (A)Line 15 indicates that serotonin comes from tryptophan. Thus the term “rate,” in “rate ofconversion” one sentence later, has to refer to the “rate at which serotonin is producedfrom tryptophan.” Logic dictates that the term cannot be defined subsequently, because it’sphrased as a back-reference. “The rate of conversion,” we read. What conversion?, we ask.Has to be the one just mentioned.

All of the other choices distort the passage. They all mention a detail in ¶ 2, but each refersto a variable that affects the rate at which tryptophan is converted into serotonin, rather thanthe rate of conversion itself.

• When questioned about scientific details, you should follow three simple rules:reread, reread, and reread. BUT: Reread the proper part of the passage, and with asense of logic. Here, there is no way that the right answer can come after line 17,because the way “rate of conversion” is introduced, it must be a follow-up to theprevious sentence.

23. (B)The crux of the passage is that an inverse relationship exists between serotonin levels in thebrain and the body’s desire for carbohydrates. When serotonin levels are low, the body’sdesire for carbohydrates is high. That’s all #23 is asking.

(A) Since insulin production promotes the production of serotonin, high levels of insulinproduction would, au contraire, lead to high serotonin levels that in turn would depress thebody’s desire for carbohydrates.

(C) is an au contraire choice, too. Serotonin is produced from tryptophan; therefore,serotonin levels will increase as the amount of tryptophan crossing the blood-brain barrierincreases, reducing the body’s desire for carbohydrates.

Page 60: 12-PrepTest 12 Explsdl.keywin.org/5/5/559e448ca144e5a6335f12b48619009d.pdf · KAPLAN LSAT PREP LSAT RELEASED TEST XII EXPLAINED A Guide to the October, 1994 LSAT KAPLAN The answer

LSAT PREP ______________________________________________________________ LSAT Test XII Explained: Section III

58 © K A P L A N

(D) Au contraire, encore! Lines 53-55 suggest that neurotransmitters other than serotoninpromote the body’s desire for carbohydrates. So, if the operation of these neurotransmitterswere to be interrupted, the body’s desire for carbohydrates would be depressed.

(E) The absorption of non-tryptophan amino acids by peripheral tissues promotestryptophan’s conversion into serotonin, which, of course, depresses the body’s desire forcarbohydrates. Au contraire, le quatrième fois.

• A solid grasp of the passage’s main idea will often help you to answer non-globalquestions, too. We saw it in #20 and we see it here.

• Pardon our French, but you need to recognize that when a passage hinges as muchas this one does on an up/down relationship, the questions are flatly going to checkwhether you’ve got it right, no ambiguity. Check out the very next question, mesamies.

24. (A)Serotonin (have we all got it by now?) reduces the body’s desire for carbohydrates, so testsubjects given a drug that inhibits serotonin’s function would naturally tend to crave foodsthat are rich in carbohydrates, rather than foods that are rich in substances like protein.

(B) People who crave carbohydrates would feel refreshed after eating a carbohydrate-filledmeal. People who don’t crave carbohydrates, in contrast, would feel sleepy.

(C) Wrong again. As lines 13-14 and 54-55 indicate, desire for carbohydrates and weightgain go hand-in-hand.

(D) Wrong again. If the amount of serotonin produced by the body is inhibited by a drug,then blood tryptophan levels, which determine serotonin levels, would also likely decreaseas a consequence of the drug’s affects.

(E) Wrong, wrong, wrong! The increased desire for carbohydrate-rich foods comes at theexpense of foods that are rich in substances like protein.

• In questions that test your understanding of a relationship, avoid choices that getthe relationship backwards. As we have now seen ad nauseam, the test makers areespecially fond of au contraire choices in these questions.

25. (E)¶ 2 discusses the process by which tryptophan is converted into serotonin; and, as line 17-19 point out, this process “is affected by the proportion of carbohydrates in an individual’sdiet...”

(A) refers to the purpose of ¶ 3.

(B) focuses on the content of ¶ 2, not its purpose.

(C) focuses on a detail in ¶ 1 (lines 3-8).

Page 61: 12-PrepTest 12 Explsdl.keywin.org/5/5/559e448ca144e5a6335f12b48619009d.pdf · KAPLAN LSAT PREP LSAT RELEASED TEST XII EXPLAINED A Guide to the October, 1994 LSAT KAPLAN The answer

LSAT PREP ______________________________________________________________ LSAT Test XII Explained: Section III

© K A P L A N 59

(D) focuses on a different detail in ¶ 1 — a question (lines 1-2) for which there is not yet aconclusive answer (lines 2-3).

• Logic questions ask how or why the author’s done something — cited a detail,included a ¶, made a reference, etc. When you’re faced with this type of question, besure to keep the author’s purpose in mind. Ask yourself not what the content issaying, but why the content is there at this point in the passage.

26. (C)Lines 46-51 say that d-fenfluramine is a serotonin-like drug that suppresses carbohydrateconsumption. It accomplishes this task by releasing serotonin into the brain, which, in turn,suppresses the body’s desire for carbohydrates. Another straightforward question askingdirectly about a straightforward relationship.

(A) and (E) are both au contraire: Consumption of carbohydrates is associated with weightgain, but this drug suppresses their intake (A). And lines 19-20 indicate that the amount ofinsulin produced by the body increases as carbohydrate intake increases. Since d-fenfluramine decreases carbohydrate intake, the amount of insulin produced by someoneon this drug would decrease, not increase (E).

(B) and (D) distort the passage. The passage doesn’t say what, if any, effect this drug wouldhave on overall sleeping patterns. The only thing we can infer about a connection betweenthe drug and sleeping behavior is that people taking the drug would become drowsy ifthey ate a carbohydrate-rich meal, because the drug suppresses an individual’s desire forcarbohydrates.

• In inference questions, watch for choices such as (B) and (D) that employ thepassage’s language but distort its ideas. Avoid them.

27. (E)The first sentence of the passage raises a question about a phenomenon (the link betweenserotonin levels and carbohydrate intake). The second sentence says that the question canbe answered only in part. The remainder of the passage then goes on to provideinformation about the part of the phenomenon that can be explained.

(A), (B), (C), and (D) use the wrong verbs. The author simply presents the current state ofknowledge about a phenomenon; he’s not out to defend, correct, assess, or suggest anything.All of these choices imply that he voices an opinion of his own — but that’s simply not thecase.

• In primary purpose questions, start off with a “verb scan” to eliminate choiceswhose verb is clearly off.

Page 62: 12-PrepTest 12 Explsdl.keywin.org/5/5/559e448ca144e5a6335f12b48619009d.pdf · KAPLAN LSAT PREP LSAT RELEASED TEST XII EXPLAINED A Guide to the October, 1994 LSAT KAPLAN The answer

60 © K A P L A N

SECTION IV:

LOGICAL REASONING

Page 63: 12-PrepTest 12 Explsdl.keywin.org/5/5/559e448ca144e5a6335f12b48619009d.pdf · KAPLAN LSAT PREP LSAT RELEASED TEST XII EXPLAINED A Guide to the October, 1994 LSAT KAPLAN The answer

LSAT PREP ______________________________________________________________ LSAT Test XII Explained: Section IV

© K A P L A N 61

1. (D)The reason most regular coffee drinkers prefer arabica to robusta is arabica's superiorrichness. Those who switch to decaffeinated coffee prefer robusta, because it is not asaffected by decaffeination: Apparently decaffeination removes arabica's richnessadvantage. We're told that decaffeination reduces a bunch of flavor-neutral substances andone substance that affects richness. The most obvious conclusion is that decaffeinationdoesn't reduce that substance in robusta beans as much as it does in arabica beans.

(A) introduces useless background information. Questions of supply and demand areirrelevant; the argument only discusses the question of taste preferences.

(B) distorts the issue, which is taste. The argument explores how decaffeination affects therichness of coffee, which is linked to one particular substance. Although decaffeinationaffects arabica's richness more than robusta's, that doesn't mean that arabica beans arehigher in caffeine.

(C) is unwarranted. We have no idea what decaffeinated coffee drinkers think of thedifferent brands of regular coffee. The point of the argument is that decaffeination affectsarabica more than robusta, not that decaffeinated coffee drinkers are among the few whosimply like robusta beans better.

(E) deals with the motives of those who switch from regular arabica to decaffeinatedarabica, and there's no evidence to support any claim about motives.

• Beware of inference-question choices that make comparisons between groups thataren’t compared in the stimulus (e.g. the number of decaffeinated coffee drinkerswho prefer robusta regular, vs. the number of decaffeinated coffee drinkers whoprefer arabica regular).

• Follow the direction of the argument in inference questions. Doing so makes it easierto prephrase an answer. Here the key concept is “richness,” which the author relatesfirst to decaffeination, and then to one particular substance affected bydecaffeination.

2. (E)As you read the stimulus, you should have noticed the shift in scope between evidenceand conclusion. Evidence: the proportion of high school grads who go on to college isincreasing. Conclusion: the guidance counselors’ efforts to convince high schoolers tochoose careers requiring college degrees have succeeded. (E) bridges this gap; the counselorsassume that a lot of those high school grads who are in college are there to prepare forcareers requiring degrees — in other words, that students have taken the counselors’advice.

(A) We don’t know, or care, what happened to graduates during the 10-year period. Thefigures mentioned in the study are from last year and 10 years ago; the time in between isirrelevant. Even worse, (A) mentions the number of grads, but since the stimulus limitsitself to percentages we have no idea about numbers at all.

Page 64: 12-PrepTest 12 Explsdl.keywin.org/5/5/559e448ca144e5a6335f12b48619009d.pdf · KAPLAN LSAT PREP LSAT RELEASED TEST XII EXPLAINED A Guide to the October, 1994 LSAT KAPLAN The answer

LSAT PREP ______________________________________________________________ LSAT Test XII Explained: Section IV

62 © K A P L A N

(B) goes too far. By pushing careers that require college degrees, the counselors evidentlybelieve that some college courses will improve a kid’s career prospects, but they needn’tassume that any college courses will.

(C) is inconsistent with the stimulus. If the counselors believe their efforts have beensuccessful, why would they assume that some kids go to college even though they werenever advised by a counselor to do so? Sure, it’s possible that this happened, but it’s notnecessary to their argument.

(D), like (A), confuses percents and numbers. Although the counselors assume there’s beena decrease in the percentage of grads who go on to college without career plans, the numberneed not have decreased.

• With assumption questions, always compare carefully the terms in the evidence tothe terms in the conclusion. The correct assumption will fill in the gap.

• Eliminate choices inconsistent with the argument.

• Be on the lookout for answer choices that confuse percents and numbers; thisconfusion is one of the testmakers’ favorite types of wrong answers.

3. (E)Insectivorous plants eat bugs; non-insectivorous plants don't. Although their mineralrequirements are practically the same, insectivorous plants can survive in soil that is toomineral-poor to support non-insectivorous plants. The obvious suspicion is thatinsectivorous plants get their minerals from the bugs they eat.

(A) Nothing in the stimulus suggests that the bugs that make up insectivorous plants' dietare more common in mineral-poor soil than anywhere else.

(B) is a distortion. The stimulus did state that insectivorous plants can survive in soil toomineral-poor to support non-insectivorous plants, but there was no suggestion that this isthe only place they can survive.

(C) goes against the stimulus by distinguishing between the types of minerals required bythe two plant types. The stimulus said their mineral requirements were not noticeablydifferent.

(D) makes a comparison that's not sustained by the stimulus. We're told that in this oneparticular environment only insectivorous plants can survive, but that's not grounds forthe general claim that they can survive in a greater number of environments.

• It's common for wrong choices in inference questions to draw too broad aconclusion when the stimulus is actually quite narrow in scope. Don't be tempted bychoices like (A) or (D) that sound vaguely plausible or merely compatible with thestimulus. Here the correct answer never strays from the narrow ground of howinsectivorous plants get their minerals in mineral-poor ground.

Page 65: 12-PrepTest 12 Explsdl.keywin.org/5/5/559e448ca144e5a6335f12b48619009d.pdf · KAPLAN LSAT PREP LSAT RELEASED TEST XII EXPLAINED A Guide to the October, 1994 LSAT KAPLAN The answer

LSAT PREP ______________________________________________________________ LSAT Test XII Explained: Section IV

© K A P L A N 63

4. (B)Water use restrictions aren’t necessary, the argument goes, because they weren’t necessaryten years ago, and the supply of water now is the same as it was then. The assumptionmade is that the demand for water is no higher now than it was ten years ago. But if thepopulation has tripled since then, this assumption (and therefore the entire argument) isinvalidated.

(A) and (C) strengthen the argument, because both imply that there is even more wateravailable now then there was ten years ago.

(D) The issue of water-consuming home appliances is irrelevant, since we don’t knowwhat proportion of the households in the region use what type of appliance.

(E) shifts the scope of the argument from the availability of water to the price of water. Itprovides mild support for the imposition of water-use restrictions, but it doesn’t addressthe logic used in the stimulus.

• By reading the question stem first, you know that you’re dealing with a weaken theargument question. As you read the stimulus, be skeptical. There’s something wrongwith this picture. If you’re looking carefully, you’ll find it.

• An argument that justifies a present decision based on a past decision necessarilyassumes that the conditions then were more or less equivalent to the conditions now.To strengthen the argument, show the similarity between past and present. Toweaken it, show the difference.

5. (A)The argument’s response to the archaeologists’ complaint that Montgomery’s views are tooextreme is the lame claim that some professional archaeologists have views that are just asextreme. But that response, as (A) points out, avoids the issue of whether such extremismdeserves criticism. If it does, then both Montgomery and the extremist archaeologistsshould be criticized. Two wrongs don’t make a right.

(B) Nothing in the argument suggests the archaeologists consider Montgomeryunqualified; the problem is simply that they find her views extreme.

(C) Irrelevant, since the argument never suggests that many or most archaeologists arefamiliar with Montgomery’s views.

(D) Also irrelevant. The argument doesn’t claim that most archaeologists disagree withMontgomery, so there’s no reason for it to present evidence to this effect.

(E) The argument doesn’t question the opponents’ motives for criticizing Montgomery;rather, it suggests the criticism isn’t deserved because others are guilty of the same thing.

Page 66: 12-PrepTest 12 Explsdl.keywin.org/5/5/559e448ca144e5a6335f12b48619009d.pdf · KAPLAN LSAT PREP LSAT RELEASED TEST XII EXPLAINED A Guide to the October, 1994 LSAT KAPLAN The answer

LSAT PREP ______________________________________________________________ LSAT Test XII Explained: Section IV

64 © K A P L A N

6. (B)The evidence provided is that this new drug relieves the intensity of all symptoms ofchronic fatigue syndrome. The conclusion is that chronic fatigue syndrome is caused byone virus rather than several. In drawing that conclusion, the argument must assume thatit’s more likely that the drug counteracts a single virus than that it counteracts several. Ifthe likelihood is otherwise — if it’s more likely that the drug will counteract severalviruses — then the conclusion would be to the contrary and the argument would fall apart.

(A) goes too far. Although the argument claims the symptoms are caused by the samevirus, it isn’t necessary that everyone with the syndrome share these symptoms. The samevirus may provoke different symptoms in different individuals.

(C) and (D) shift the scope. (C)’s comparison of this syndrome’s symptoms to those of othersyndromes is irrelevant; the argument concerns chronic fatigue syndrome only. (D)generalizes from the case of chronic fatigue to “most” syndromes, about which theargument says nothing and therefore makes no assumptions.

(E) Since the argument doesn’t claim this new drug cures chronic fatigue syndrome, thisisn’t assumed. It’s perfectly possible for a drug to relieve the symptoms of an ailmentwithout actually curing it.

• Keep working on your command of the Denial Test for assumption questions (aswell as inference questions). It’s your best insurance against falling for wronganswers.

7. (D)This argument makes one of the testmakers’ favorite errors of reasoning: confusingcorrelation and causation. DataCom filed more patents than its competitors; DataCom wasmore financially successful than its competitors. Therefore, the argument concludes, thefirst difference must have caused the second. As (D) points out, there isn’t enoughevidence here; it’s possible that other differences between DataCom and its competitorscaused DataCom’s comparative financial success.

(A) describes circular reasoning, in which an argument’s evidence is the same as itsconclusion. Here, the evidence is a correlation and the conclusion is a causal relationship,so there’s no circularity.

(B) The argument doesn’t confuse financial success with technological innovativeness; itclaims that because of technological innovativeness (i.e., filing more patents), DataCom isfinancially successful.

(C) introduces an irrelevant distinction. The argument’s point is that DataCom, thecompany that filed the most patents, had the most financial success. It doesn’t matter ifsome of its competitors were more successful than others.

(E) There’s no generalization here because the argument doesn’t claim that all companiesthat file more patents are more successful. The conclusion addresses the reasons forDataCom’s success only.

Page 67: 12-PrepTest 12 Explsdl.keywin.org/5/5/559e448ca144e5a6335f12b48619009d.pdf · KAPLAN LSAT PREP LSAT RELEASED TEST XII EXPLAINED A Guide to the October, 1994 LSAT KAPLAN The answer

LSAT PREP ______________________________________________________________ LSAT Test XII Explained: Section IV

© K A P L A N 65

• Read critically and always try to put arguments into real-world situations. Ifsomeone gave you the argument made in this question, would you buy it, or wouldyou ask, “How would filing more patents bring about financial success?”

• Flaw questions show you the wisdom of reading the question stem first. Right away,you know your job is to locate the error in reasoning.

• Be on the lookout for arguments that confuse correlation and causation. It’s one ofthe testmakers’ favorite types of bad arguments.

8. (C)The scholars argue that because the book contains inconsistencies, its author must haverelied on more than one source. This only makes sense if they assume (C), that there’s noone source that the author used that has the same inconsistencies as those found in thebook. If there were, then the existence of the inconsistencies would only show that theauthor had drawn from one source, not several.

(A) and (D) shift the scope in a similar way. In (A), whether authors generally try to avoidinconsistencies is irrelevant. The issue is what this author did. As for (D), we don’t know, orcare, if the author knew that inconsistencies were possible. To use the Denial Test, itwouldn’t hurt the argument if the author were unaware of this.

(B) Irrelevant. The inconsistencies needn’t be so obvious that the average reader wouldnotice them. This isn’t part of the argument, so no assumptions need be made about it.

(E) goes too far. It’s possible for the author to have relied on more than one source withoutbeing familiar with all relevant sources.

• Note the unusual wording of this assumption question stem. As you read stems anddiscover unfamiliar wording, keep in mind that they may be routine question typesthat are just a bit disguised.

9. (D)The original conclusion — signaled by “therefore” — is that neither of two distinct traits(competitiveness and rules) is a condition necessary to a general entity (games). (D) may bewritten somewhat roundabout but it’s the same thing: Neither gas fuel nor four wheels, itsays, is necessary to a car. You shouldn’t have strained much here, because the other fourconclusions fall short of the mark:

(A) concludes that one trait is necessary to being a gourmet but another is not.

(B) concludes that either one trait isn’t necessary to being a bear, or a group of experts(where do they shine in?) are wrong.

(C) concludes that one trait is necessary to being a dog.

Page 68: 12-PrepTest 12 Explsdl.keywin.org/5/5/559e448ca144e5a6335f12b48619009d.pdf · KAPLAN LSAT PREP LSAT RELEASED TEST XII EXPLAINED A Guide to the October, 1994 LSAT KAPLAN The answer

LSAT PREP ______________________________________________________________ LSAT Test XII Explained: Section IV

66 © K A P L A N

(E) concludes that two traits that Montreal does possess aren’t unique to Montreal, but areshared by other cities. (E) uses the word “essentially” in a manner different from thestimulus and other choices.

• As a first step in Parallel Logic, compare the conclusion of the stimulus to that ofeach of the choices.

10. (A)The author begins by stating the theory that household indebtedness causes recession, andthen examines whether that theory applies to “the recent recession.” He considers ahypothetical scenario in which the theory might apply, but then says that this scenario didnot obtain during the recent recession. His conclusion is that the theory doesn't apply:Household indebtedness was not the cause of the recent recession. Pretty straightforward.

(B) distorts the point about the amount of debt in low-income households. The author saidthat low-income households never had very much debt in the first place because money isonly lent to those with assets.

(C) There's no reason to think that affluent people increased their spending. In fact, sincethe author concludes that high-income households owed most of the debt, it seemsunlikely.

(D) Although the argument implies that high household indebtedness didn’t cause the laterecession, there's no implication that it has little impact on the economy; that's much toostrong.

(E) strays outside the scope by discussing what people did with the money they borrowed.That's an irrelevant issue.

• In a long and involved stimulus, it can be a particular help to keep an eye on the keywords. At line 4, “admittedly” tips you off that the section in the middle is examininga possible objection to the argument, and is certainly not the conclusion.

11. (A)The author gave one scenario in which it was possible for household debt to have causedthe recession (by decreasing spending among high-debt low-income households). He thenrejected this scenario because no one lends money to people without assets (i.e. low-income households). So he concluded that household debt wasn't responsible for therecession. But he overlooks the possibility that there is some other scenario, one that hehasn't considered, in which it's also possible for household debt to cause the recession.That's what (A) gives: middle-income families, who presumably have some assets andtherefore can accumulate debt, slowing spending because of their debt.

(B) has no effect on the argument, which rejected household debt as a cause for therecession because of the supposed distribution of debt (most owed by those with assets) notbecause of the total amount of debt.

Page 69: 12-PrepTest 12 Explsdl.keywin.org/5/5/559e448ca144e5a6335f12b48619009d.pdf · KAPLAN LSAT PREP LSAT RELEASED TEST XII EXPLAINED A Guide to the October, 1994 LSAT KAPLAN The answer

LSAT PREP ______________________________________________________________ LSAT Test XII Explained: Section IV

© K A P L A N 67

(C) First, it's not clear how much effect the decrease in (C) would have, and second, there'sno evidence that that decrease was due to household debt, which is the crux of theargument. Stick to the scope!

(D) supplies useless background information, giving a reason for the affluent to go intodebt but not showing why this would mean that their debt could cause the recession.

(E) is an au contraire choice, supporting the author's contention that household debt wasn'tresponsible for the recession.

• If you find yourself puzzling too long over what possible effect a choice could haveon the argument, that's a good sign that it’s not the credited choice.

12. (A)When we strip away all the technical jargon, it boils down to this: Carbon dioxide warmsthe atmosphere, while sulfate aerosols cool it. Both are produced by fossil-fuel emissions.If fossil fuel emissions are cut, the atmosphere will continue to get warmer for more thanthree decades before leveling off. This can be explained by the fact that the carbon dioxideproduced by fossil-fuels stays in the atmosphere for a long time, while the sulfate aerosolsdisappear quickly.

(B) shifts the scope of the argument. We’re concerned with the atmosphere of the entireplanet, not just the Northern Hemisphere.

(C) and (D) make irrelevant distinctions. The difference between how long sulfur dioxideand carbon dioxide have been thought of as pollutants (C) is irrelevant because theargument concerns the future, not the past. And the argument postulates the cutting of allfossil-fuel emissions, not just those produced by cars, so (D)’s distinction is likewise bogus.

(E) provides support for the reduction of fossil-fuel emissions, but that’s not the questionthat’s being asked.

• Read for keywords. A phrase like “yet, surprisingly,” is a tip-off that you’ll be askedto resolve an apparent paradox.

• The LSAT is designed to test your preparedness for law school, not for Caltech. Youdon’t need to understand every nuance of a scientific argument. The broad outlines(e.g. carbon dioxide - warm, sulfur - cool) should be enough to answer the question.

13. (C)The question stem asks us to “support the view that a physician’s responding to the requestwould violate medical ethics.” In other words, we want to find a principle that wouldweaken the argument in the stimulus. That’s a switch from standard LSAT practice.

It would not be unethical, the passage argues, for a doctor to respond to the wanted poster,because it’s not unethical for doctors to inform the police about gunshot wounds. So toweaken the argument, we need a choice that invalidates the analogy between the two

Page 70: 12-PrepTest 12 Explsdl.keywin.org/5/5/559e448ca144e5a6335f12b48619009d.pdf · KAPLAN LSAT PREP LSAT RELEASED TEST XII EXPLAINED A Guide to the October, 1994 LSAT KAPLAN The answer

LSAT PREP ______________________________________________________________ LSAT Test XII Explained: Section IV

68 © K A P L A N

situations. Choice (C) states that gunshot wounds are a unique exception to the generalrule that it is unethical for doctors to disclose their patients’ identities to the police. (Somemay have been troubled by the lack of reference in the right answer to infectious diseases— the other exception mentioned in the stimulus — and thus been dissuaded fromchoosing (C). Recognize that the scope of both the wanted-poster situation and choice (C) isrestricted to disclosure to law enforcement agencies only. The disease detail is tangential tothis situation.)

(A) is an au contraire choice providing strong support for the view that a doctor’sresponding to the police request is ethical.

(B) provides general support for the view that responding to the request is unethical, but itdoesn’t address the crucial issue of gunshot wounds. The question stem insists that thecorrect principle “remain compatible with the requirements cited above,” and (B) wouldrender the gunshot wound disclosure unethical.

(D) muddies the waters by introducing the question of the patient’s consent. This shifts thescope of discussion away from the core of the argument. Also, if a patient who’d been shotrefused under (D) to give consent to disclosure, then the situation in the stimulus wouldbecome unethical; but we can brook no contradiction with the stimulus in the right answer.

(E) Like Choice (B) above, this principle provides general support to the idea in question,but (also like (B), and maybe (D) as well) it leaves out the possibility that doctors couldethically report gunshot wounds to police, thus rendering the stimulus situation unethical.This choice is tempting because it evokes the infectious-disease disclosure part of thestimulus, but as noted above, that kind of disclosure is outside the scope of this question.

• Decode the question stem carefully, even if at first glance it seems 100% routine. Thisquestion asks you to “support” an idea, but actually calls for you to weaken theargument in the stimulus, and if you fail to notice that you almost certainly get thequestion wrong.

14. (D)Pure method-of-argument, with no announcement that either speaker’s argument isflawed. Ingrid’s judgment that rock songs aren’t as “durable” as ‘40’s songs is based on herdefinition of durability: songs that continue to enjoy new recordings. Jerome’s definition ofdurability — songs that remain popular even in their original incarnation — is somewhatdifferent, “an alternative standard,” to quote the correct choice. Whose definition is better?No way to tell and that’s not our job. What we do need to see is that, by Jerome’s definition,rock songs may in fact turn out to be durable.

(A) Jerome has misinterpreted nothing, intentionally or otherwise; he has reinterpreted akey term.

(B) There’s no contradiction anywhere here: Jerome doesn’t say or prove that Ingrid iscontradicting herself, and he doesn’t so much contradict her as point out another way that“durability” can fairly be interpreted.

Page 71: 12-PrepTest 12 Explsdl.keywin.org/5/5/559e448ca144e5a6335f12b48619009d.pdf · KAPLAN LSAT PREP LSAT RELEASED TEST XII EXPLAINED A Guide to the October, 1994 LSAT KAPLAN The answer

LSAT PREP ______________________________________________________________ LSAT Test XII Explained: Section IV

© K A P L A N 69

(C) Au contraire, he agrees with her and affirms her evidence. What they disagree on is thenecessary meaning of “durability.”

(E) What period? Jerome’s talking about the future durability of rock songs; he doesn’teven bring up the ‘40’s.

• As a rule, reject choices like (A), that impugn someone’s motives. Usually we aren’tgiven enough information to adjudge intentionality, and even if we were it wouldgenerally be beside the point of the logic.

15. (B)The argument shows that doctors have both the incentive and the opportunity to performunnecessary medical treatment, and from there fallaciously concludes that they must do so.As many of us know from murder mysteries and police procedurals, just because asuspect has motive and opportunity doesn’t prove that he or she dunnit. Here, the doctorsdidn’t necessarily dunnit.

(A) It’s hard to say that a doctor’s involvement in the events leading to a patient receivingmedical treatment is “purely coincidental.”

(C) We’d like to be able to rise to the defense of the medical profession, but we have noevidence in this case to suggest that all medical decisions “are based on the rigorousapplication of well-defined principles.”

(D) The argument doesn’t suggest that doctor’s decisions to perform medical treatmentsare arbitrary. It states pretty flatly that those choices are financially motivated.

(E) The stimulus does suggest that expense is an irrelevant consideration for the patient,but it doesn’t suggest that expense is irrelevant to the doctor. On the contrary, it says flat outthat the expense is the doctor’s motivating force.

• For a technique or method of argument question, don’t worry about whether theargument is wrong or right. Instead, think about how the argument works. Theargument may be illogical, but why is it illogical?

16. (E)If chlorofluorocarbons are the best possible solvents for cleaning electronic sensors, and ifthese solvents are very important in helping cars meet admission standards, then phasingout chlorofluorocarbons creates a problem. Unless cars simply fail to meet the new stricterstandards, something else is going to have to take up the slack. We're told to assume thatcars continue to meet the standards, so we can conclude that, now that solvents aren't aseffective as they were, something else is going to take on a greater importance in helpingcars meet admission standards.

(A) We can't conclude anything about what attitude automakers will adapt towards oneanother. Maybe competition will become more fierce.

Page 72: 12-PrepTest 12 Explsdl.keywin.org/5/5/559e448ca144e5a6335f12b48619009d.pdf · KAPLAN LSAT PREP LSAT RELEASED TEST XII EXPLAINED A Guide to the October, 1994 LSAT KAPLAN The answer

LSAT PREP ______________________________________________________________ LSAT Test XII Explained: Section IV

70 © K A P L A N

(B) is too strong a conclusion. Although solvents for sensors won't be as important inmaintaining admissions standards, we don't know that sensors will therefore be abandoned(we don't even know if this is possible).

(C) introduces an entirely new idea (smaller cars vs. larger cars) and also a subtle scopeshift. The problem is meeting emissions standards, and this is not necessarily the samething as keeping gas mileage down. A fuel-efficient car may still emit many pollutants.

(D) isn't necessarily true. Use the Denial Test: Even if the new solvents are far less effective,some other factor or factors could do enough to ensure that admissions standards are stillmet (as (E) says).

• Be suspicious of inference choices (like (B) above) that exhibit too much certaintywhen the stimulus is phrased in less absolute terms. Here, the stimulus is qualified(“solvents have contributed significantly”) so the credited choice is similarlymoderate (“something...will have to make a relatively greater contribution”).

17. (C)The studies cited show that drug Z is at best only slightly more effective than drug Y atpreventing blood clots. The argument concludes that there is no established medicalreason to use Z instead of Y. As (C) points out, this is a bit of a leap, as there may be othermedical reasons for choosing a particular drug besides its relative effectiveness. Forexample, if a patient were allergic to drug Y, the doctor would have an excellent medicalreason for using drug Z instead.

(A) Irrelevant. The argument limits its evidence and conclusion to a comparison of drugsY and Z. Other drugs are outside the scope.

(B) and (E) play off the red herring about Z’s manufacturer’s questionable marketingpractices. The argument against Z is based solely on its medical effectiveness, so contraryto (B), there’s no need to bring in Y’s manufacturer’s marketing practices. As for (E), there’sno confusion of economic and medical reasons; medical reasons are the only ones given fornot using Z.

(D) The chemical composition of neither drug is mentioned, so the argument doesn’t makeany such assumption.

• The longer the stimulus, the more likely it is that it contains “red herrings” (like themarketing practices here) designed to throw you off the scent. Stay focused on theessential argument structure of evidence and conclusion.

Page 73: 12-PrepTest 12 Explsdl.keywin.org/5/5/559e448ca144e5a6335f12b48619009d.pdf · KAPLAN LSAT PREP LSAT RELEASED TEST XII EXPLAINED A Guide to the October, 1994 LSAT KAPLAN The answer

LSAT PREP ______________________________________________________________ LSAT Test XII Explained: Section IV

© K A P L A N 71

18. (E)Since the claim against Z was that there was no medical reason to recommend it, we wantto strengthen the case for Z with a principle that provides a medical reason. (E) plays offthe fact that the studies showed that Z was “no more or only slightly more” effective than Y.The belief that any chance one drug is better than the other justifies using the possiblybetter one regardless of cost would certainly justify a doctor’s decision to choose the moreexpensive Z over Y.

(A) shifts the scope to the issue of cost, which is only mentioned as an aside in the originalargument. (A)’s point is that doctors should only use more expensive drugs on patientswho can afford them. At best, this might give doctors a reason why they needn’t avoid usingZ with certain patients, but such a halfhearted recommendation doesn’t help Z’s casenearly as much as (E) does. Remember, we want the principle that most justifies using Zover Y.

(B) Irrelevant. This principle might justify Z’s manufacturer’s marketing practices, but thatisn’t what we’re asked to do.

(C) Irrelevant, since the argument wasn’t based on marketing practices to begin with.Notice that this choice is similar to (B) in question 17.

(D) Irrelevant. Z’s manufacturer doesn’t even contest the studies’ results, and even if itdid, this principle would only justify rejecting its criticism, not picking Z over Y.

• Avoid skipping stimuli that come with two questions. Once you’ve invested time inanswering one of the questions, the other one often comes easily.

• The answer to principle questions is usually the choice that expresses the keyconcepts and terms left out by the other choices. Here, (E) is the only choice thatdirectly refutes the medical reasons for rejecting drug Z.

19. (B)(B) is pretty clear-cut in describing Alan’s error. Jane’s argument is certainly undermined— that is, the link between evidence and conclusion is weakened if not outright severed —when Alan shows that her evidence (the article about infants under 3) is inapplicable totheir older kids. Certainly, the article isn’t adequate evidence for the conclusion that thesekids’ TV viewing should be restricted. Nevertheless, the conclusion itself could still betrue — it still could be a good idea to restrict Jacqueline and Mildred’s TV, albeit fordifferent reasons — yet Alan denies that.

(A) It’s not a flaw to assess your opponent’s evidence in a debate. In fact, it’s commonpractice; after all, you can only attack your opponent’s argument on his or her own terms,and those terms include the original evidence. Beyond that, Alan doesn’t “rely” on thearticle so much as reevaluate it.

Page 74: 12-PrepTest 12 Explsdl.keywin.org/5/5/559e448ca144e5a6335f12b48619009d.pdf · KAPLAN LSAT PREP LSAT RELEASED TEST XII EXPLAINED A Guide to the October, 1994 LSAT KAPLAN The answer

LSAT PREP ______________________________________________________________ LSAT Test XII Explained: Section IV

72 © K A P L A N

(C) Clearly he does address her main point (“Should our daughters’ TV be restricted?”).And just as clearly, his reference to the ages is germane. In a way, Alan is accusing Jane ofcommitting a “scope shift” — the scope of the article is the effect of TV on infants, and sheis applying it to her own, older children. A debate on the scope of an argument is hardly a“side issue.”

(D) Alan’s point is not “irrelevant” [see discussion of (C), above]. Moreover, an “appeal toauthority” is essentially arguing that “my conclusion is true because this expert says so,”but Alan isn’t saying that.

(E) accuses Alan of essentially confusing cause and effect — of mistaking the one for theother. But in no way is he suggesting that poor hand-eye coordination leads to all of thatexcess TV watching. You might have been attracted by the sound of (E), but we hope youwere repelled by its smell.

• This question reminds us of a fundamental point: To weaken an argument is to severor weaken the link between evidence and conclusion, and need not mean that theconclusion has been proven false.

• Don’t miss references to “reasoning errors” in question stems. It’s convenientwhenever the testmakers tell you whether reasoning is strong or faulty.

20. (A)This stimulus contains a classic scope shift: The evidence limits itself to compression stresson the spine, while the conclusion leaps to the more general topic of the risk of spinalinjury. (A) bridges this gap: The argument only holds water if compression stress frompushing is the only cause of spinal injury from raking. If there are other sources of spinalinjury, then the author would have no reason to recommend straight-handled rakes.

(B) Irrelevant, as frequency of injury isn’t part of the argument. Even if injury from rakingoccurs only rarely, the author is still justified in recommending straight-handled rakes,provided (A)’s assumption is true.

(C) Irrelevant. We’re not interested in what happens generally when a tool is redesigned;we’re interested in what happened in this instance. Even if, contrary to (C), tool redesign isusually successful, that doesn’t weaken the author’s argument that it wasn’t successfulhere.

(D) Equally irrelevant. If (D) were false and one could use a rake with only push strokes,the author’s argument in favor of straight-handled rakes wouldn’t be affected.

(E) introduces an irrelevant comparison. Who cares if there are other possible rakedesigns? The argument compares straight-handled rakes to rakes with S-shaped handles;other handle shapes are beyond the scope.

• The correct assumption is the one that fills in the missing link between evidence andconclusion. Use the Denial Test to check your answers to assumption questions.

Page 75: 12-PrepTest 12 Explsdl.keywin.org/5/5/559e448ca144e5a6335f12b48619009d.pdf · KAPLAN LSAT PREP LSAT RELEASED TEST XII EXPLAINED A Guide to the October, 1994 LSAT KAPLAN The answer

LSAT PREP ______________________________________________________________ LSAT Test XII Explained: Section IV

© K A P L A N 73

21. (D)If the ice caps melt, then the polar seas will warm. Since the seas haven’t begun to warm,say the scientists, the melting of the ice-caps hasn’t started. But if it were true that watertemperature tends to remain constant until all the ice has melted, then the fact that the seashaven’t warmed doesn’t necessarily mean that the ice-caps haven’t started to melt.

(A) What happens to the plants should the ice-caps melt has no bearing on the discussionof whether they are actually melting.

(B) would have it that the scientists’ conclusion is erroneous in that they should be lookingat water levels instead of water temperature. But this doesn’t provide any evidence thatwater temperature isn’t a reliable indicator of whether the ice is melting.

(C) is a scope-shift. Since the scientists’ conclusion is based on the water temperature,information about the air temperature doesn’t affect the argument.

(E) is beyond the scope. We don’t have any reason to entertain the notion that thetemperature of the equatorial seas is related to that of the polar seas.

• Be aware of minor scope shifts, especially if an argument seems particularly logical.The scientists’ argument says, in effect, that something hasn’t started because there’sno evidence that it’s been completed. Choice (D) undermines their conclusion bypointing out the flaw in this scope shift.

Page 76: 12-PrepTest 12 Explsdl.keywin.org/5/5/559e448ca144e5a6335f12b48619009d.pdf · KAPLAN LSAT PREP LSAT RELEASED TEST XII EXPLAINED A Guide to the October, 1994 LSAT KAPLAN The answer

LSAT PREP ______________________________________________________________ LSAT Test XII Explained: Section IV

74 © K A P L A N

22. (C)The study found that those people who gained weight from age 35 onward tended to livelonger than those who didn’t gain weight. This could be explained by the fact that smokers,who are less likely to gain weight, tend to have shorter life spans.

(A) is useless background information. We don’t know enough about physiology todetermine if it has any impact on the argument.

(B), (D), and (E) all support the “other studies that have associated weight gain with a hostof health problems that tend to lower life expectancy.” They don’t, however, help us toresolve the discrepancy between these “other studies” and the data in question.

• A paradox question presents you with two conflicting opinions, both of which seemto be supported by the facts. The answer won’t be as simple as: one opinion is right,the other is wrong. The correct answer choice will present new facts that explain orclarify the situation.

• All surveys and experiments are limited. You just can’t study everything at once. Soif a paradox question involves a scientific study, think about what the study didn’ttake into account that might be relevant. In this case, what other factors besidesweight affect health and life expectancy?

23. (C)More parallel logic, and once again, first zeroing in on the conclusion reduces your time topractically nothing. The stimulus conclusion “X does not bring about Y” is essentially acategorical statement of negative cause and effect, just as in (C). What do we get in thewrong choices?

(A) “X is typically found in Y” — a correlation.

(B) “Most X contain Y” — another correlation.

(D) “X usually have Y” — yet another. All three of these choices are too qualified, andneglect cause and effect altogether.

(E) “If you want X, you should first do Y” — a precondition to a policy.

• Don’t split hairs in Parallel Logic.

Page 77: 12-PrepTest 12 Explsdl.keywin.org/5/5/559e448ca144e5a6335f12b48619009d.pdf · KAPLAN LSAT PREP LSAT RELEASED TEST XII EXPLAINED A Guide to the October, 1994 LSAT KAPLAN The answer

LSAT PREP ______________________________________________________________ LSAT Test XII Explained: Section IV

© K A P L A N 75

24. (A)According to environmental economists, it’s impossible to compare environmental factorswith monetary costs and benefits to make an economic decision. So environmentaleconomists must assign monetary values to environmental factors. But we typically assignsomething a monetary value by comparing it with monetary costs and benefits and makingan economic decision, which puts us right back where we started. So the argument’sconclusion, that environmental economics can’t work for the very reason that it’s needed, isa valid one, if we assume that there’s no other way to assign monetary value to these factors.

(B) says that the passage’s conclusion is strongly supported, “unless” economic decision-making doesn’t effect the environment. But what if it were true that economic decisionsdidn’t affect environmental factors? It might question the need for environmentaleconomics, but it wouldn’t undermine the argument’s conclusion, that environmentaleconomics “is stymied by what motivates us.”

(C) The heart of the argument isn’t that environmental economics is necessary, it’s thatenvironmental economics doesn’t work. So there’s no need to establish that economicdecision-makers don’t take account of environmental factors.

(D) raises the issue of whether economic decision-making affects the environment. This isbeyond the scope of the argument, which deals only with how environmental factors affecteconomic decisions.

(E) The passage does involve circular reasoning. But the conclusion is valid, because itsays, in effect, that this circular reasoning is the problem faced by environmentaleconomics.

• Use Keywords. Understanding the significance of words like “but” and “thus” canhelp you to make sense out of a dense or convoluted passage.

• When dealing with critique-the-logic questions, you may need no more than the firstword or two of an answer choice in order to eliminate it. Here, there were really onlythree types of answer choice: The passage’s conclusion is supported strongly,weakly, or not at all. If you made that decision first, you could have eliminated threeor four choices without even reading them all the way through.

Page 78: 12-PrepTest 12 Explsdl.keywin.org/5/5/559e448ca144e5a6335f12b48619009d.pdf · KAPLAN LSAT PREP LSAT RELEASED TEST XII EXPLAINED A Guide to the October, 1994 LSAT KAPLAN The answer

I.N. LL3101 REV. A PRINTED IN USA